Download as pdf or txt
Download as pdf or txt
You are on page 1of 45

Page 1 of 45

PTS 2024 | B4 | L1 Test 2 - Solutions |

Q.1)
Ans) b
Exp) Option b is the correct answer.
Departmentally Related Standing Committees (DRSCs) have been set up to secure more accountability of
the Executive to Parliament, particularly financial accountability.
Each Departmentally Related Standing Committee consists of 31 members (21 from Lok Sabha and 10 from
Rajya Sabha). The members from the Lok Sabha are nominated by the Speaker, just as the members from
the Rajya Sabha are nominated by the Chairman from amongst its members. A minister is not eligible to
be nominated as a member of any of the standing committees.
Statement 1 is incorrect: They are not constitutional bodies. On the recommendation of the Rules
Committee of the Lok Sabha, 17 DRSCs were set up in Parliament in 1993. In 2004, seven more such
committees were set up, thus increasing their number from 17 to 24. Out of the 24 Departmental
Standing Committees, 8 work under the Rajya Sabha and 16 under the Lok Sabha.
Statement 2 is incorrect: The term of office of each standing committee is one year from the date of its
constitution.
Statement 3 is correct: They secure more accountability of the Council of Ministers to the Parliament.
DRSCs secure financial accountability on the Executive and assists the Parliament in debating the budget
more effectively.
Statement 4 is correct: The Committees aid and assist the Legislature in discharging its duties and
regulating its functions effectively, expeditiously and efficiently. They assist the Parliament in
thoroughly and systematically scrutinizing the bills and matters which could not be discussed on the
floor at length. These bills are referred to the Committee by the Chairman or the Speaker. The
Committees also provide the expertise on a matter which is referred to them.
Knowledge Base:
These DRSCs are entrusted with the following functions: -
(a) to consider the Demands for Grants of the related Ministries/Departments and report thereon. The
report shall not suggest anything of the nature of cut motions;
(b) to examine Bills, pertaining to the related Ministries/Departments, referred to the Committee by the
Chairman or the Speaker, as the case may be, and report thereon;
(c) to consider the annual reports of the Ministries/Departments and report thereon; and
(d) to consider national basic long term policy documents presented to the Houses, if referred to the
Committee by the Chairman or the Speaker, as the case may be, and report thereon.
Source: Laxmikant C-23
https://rajyasabha.nic.in/Committees/DepartmentRelatedSC_RS?id=17

Q.2)
Ans) c
Exp) Option c is the correct answer
As the office of the Speaker is vested with great prestige, position and authority, independence and
impartiality becomes essential for the proper functioning of democracy in India.
Statement 1 is incorrect: Unlike in Britain where the Speaker has to resign from his/her party and
remain politically neutral, in India the Speaker does not resign from his/her party after being elected to
the office of Speaker.
Statement 2 is correct: The salaries and allowances of the Speaker are charged on the Consolidated Fund
of India, thus it cannot be subjected to the vote of Parliament. Also, the salaries and allowances of the
Speaker are fixed by the Parliament. It ensures the independence of the Speaker of Lok Sabha.

Forum Learning Centre: Delhi - 2nd Floor, IAPL House, 19 Pusa Road, Karol Bagh, New Delhi - 110005 | Patna - 2nd floor, AG Palace, E Boring Canal
Road, Patna, Bihar 800001 | Hyderabad - 1st & 2nd Floor, SM Plaza, RTC X Rd, Indira Park Road, Jawahar Nagar, Hyderabad, Telangana 500020
9311740400, 9311740900 | https://academy.forumias.com | admissions@forumias.academy | helpdesk@forumias.academy
Page 2 of 45

PTS 2024 | B4 | L1 Test 2 - Solutions |

Statement 3 is correct: It is true that the Speaker's power of conducting business is beyond the
Jurisdiction of any Courts. This provision enables the Speaker to carry out his/her role without any fear
and ensures the independence and impartiality of the office of Speaker.
Statement 4 is correct: It is true that unless a substantive motion is passed, the Speaker’s work and
conduct in the house cannot be discussed and criticised in the Lok Sabha. Substantive motion is a self-
contained independent proposal submitted for the approval of the House and drafted in such a way as to
be capable of expressing a decision of the house.
Source: Laxmikanth - Chapter 22 (Parliament)

Q.3)
Ans) c
Exp) Option c is the correct answer.
The Seventh Amendment, 1956 to the Indian Constitution brought about the abolition of the previous
classification of states into Part A, Part B, Part C, and Part D states. Instead, it led to the reorganization of
the States into 14 States and 6 Union Territories. This amendment aimed to streamline and simplify the
administrative divisions of the country, ensuring better governance and effective representation for the
states and union territories.
Source: Laxmikanth, 6th edition, Appendix IV Constitutional Amendments at a Glance

Q.4)
Ans) b
Exp) Option b is the correct answer.
Statement 1 is correct: The Member of Parliament takes the oath of office to
1) bear true faith and allegiance to the Constitution of India
2) uphold the sovereignty and integrity of India; and
3) faithfully discharge the duty upon which he is about to enter.
Statement 2 is incorrect: Members of Parliament (MP) take an oath before the President of India (not
Presiding officer of the house) or some person appointed by him for this purpose.
Statement 3 is correct: It is true that unless a Member of Parliament (MPs) take an oath, he/she cannot
vote and participate in the proceedings of the House and does not become eligible to parliamentary
privileges and immunities.
Source: Laxmikanth - Chapter 22 (Parliament)

Q.5)
Ans) d
Exp) Option d is the correct answer.
Recently, once nearly extinct species of Himalayan Griffon spotted in Dudhwa & Banbasa and seen
nesting in Terai.
Statement 1 is incorrect: Himalayan Griffon is an Old-World vulture in the family Accipitridae. It is one
of the two largest Old-World vultures and true raptors. This species is found along the Himalayas and the
adjoining Tibetan Plateau. This species is distributed from western China, Kazakhstan, Uzbekistan,
Kyrgyzstan, Tajikistan, Afghanistan and Pakistan, east through the Himalayan Mountain range in India,
Nepal and Bhutan, to central China and Mongolia.

Forum Learning Centre: Delhi - 2nd Floor, IAPL House, 19 Pusa Road, Karol Bagh, New Delhi - 110005 | Patna - 2nd floor, AG Palace, E Boring Canal
Road, Patna, Bihar 800001 | Hyderabad - 1st & 2nd Floor, SM Plaza, RTC X Rd, Indira Park Road, Jawahar Nagar, Hyderabad, Telangana 500020
9311740400, 9311740900 | https://academy.forumias.com | admissions@forumias.academy | helpdesk@forumias.academy
Page 3 of 45

PTS 2024 | B4 | L1 Test 2 - Solutions |

Himalayan Griffon
Statement 2 is incorrect: Himalayan vultures are susceptible to toxicity induced by diclofenac, a drug
whose residues are in domestic animal carcasses. But their population have however not shown signs of
rapid decline as witnessed in populations of other Gyps vultures across Asia.
Statement 3 is incorrect: According to the International Union for Conservation of Nature (IUCN), the
Himalayan Griffon (Gyps himalayensis) is classified as "Near Threatened" on the IUCN Red List of
Threatened Species. This means that the species is at risk of becoming threatened with extinction in the
near future if the current conservation efforts are not maintained.
Knowledge Base: Old World vultures are a group of vultures that are found in Europe, Africa, and Asia.
They belong to the family Accipitridae, which also includes eagles, hawks, and kites. Old World vultures
are different from New World vultures, which are found in the Americas, in that they have a hooked beak,
rather than a straight one, and their nostrils are not divided by a septum.
Source:https://timesofindia.indiatimes.com/city/dehradun/almost-extinct-vulture-spotted-in-
terai/articleshow/98221968.cms?from=mdr

Q.6)
Ans) c
Exp) Option c is the correct answer.
Pair 1 is correct: Summoning implies the starting of a session of Parliament. The President from time to
time summons each House of Parliament to meet. The maximum gap between two sessions
of Parliament cannot be more than six months i.e, the Parliament should meet at least twice a year.
Pair 2 is incorrect: An Adjournment suspends the sitting of Parliament for a specified time, which may
be hours, days or weeks. It only terminates a sitting and not a session of the House.
Pair 3 is correct: Adjournment sine die means terminating a sitting of Parliament for an indefinite
period. The power of adjournment as well as adjournment sine die lies with the presiding officer of the
House
Pair 4 is correct: Prorogation not only terminates a sitting but also a session of the House. Unlike an
adjournment or adjournment sine die, the President of India issues prorogation ending the session of
Parliament.
Source: Laxmikanth - Chapter 22 (Parliament)

Q.7)
Ans) a
Exp) Option a is the correct answer.
Adjournment Motion interrupts the normal business of the House, it is regarded as an extraordinary
device. The discussion on an adjournment motion should last for not less than two hours and thirty
minutes.

Forum Learning Centre: Delhi - 2nd Floor, IAPL House, 19 Pusa Road, Karol Bagh, New Delhi - 110005 | Patna - 2nd floor, AG Palace, E Boring Canal
Road, Patna, Bihar 800001 | Hyderabad - 1st & 2nd Floor, SM Plaza, RTC X Rd, Indira Park Road, Jawahar Nagar, Hyderabad, Telangana 500020
9311740400, 9311740900 | https://academy.forumias.com | admissions@forumias.academy | helpdesk@forumias.academy
Page 4 of 45

PTS 2024 | B4 | L1 Test 2 - Solutions |

Statement 1 is incorrect: Adjournment Motion involves an element of censure against the government
and hence Rajya Sabha is not permitted to make use of this device. Adjournment motion can be moved
only in Lok Sabha.
Statement 2 is incorrect: Adjournment Motion is introduced in Lok Sabha to draw attention of the
House to a definite matter of urgent public importance and needs the support of 50 members to be
admitted.
Statement 3 is correct: The right to move a motion for an adjournment of the business of the House is
subject to the following restrictions:
1) It should not revive discussion on a matter that has been discussed in the same session.
2) The adjournment motion should not raise a question of privilege.
3) It should not cover more than one matter.
4) It should raise a matter which is definite, factual, urgent and of public importance.
Source: Laxmikanth chapter 22 – PARLIAMENT, page 512.

Q.8)
Ans) d
Exp) Option d is the correct answer.
The Parliament is the legislative organ of the Union government. It occupies a pre-eminent and central
position in the Indian democratic political system due to adoption of the parliamentary form of
government, also known as ‘Westminster’ model of government.
Statement 1 is incorrect: Under the Constitution, the Parliament of India consists of three parts viz, the
President, the Council of States and the House of the People. The President is an integral part of the
Parliament. This is because a bill passed by both the Houses of Parliament cannot become law without
the President’s assent. He also performs certain functions relating to the proceedings of the Parliament,
for example, he summons and pro-rogues both the Houses, dissolves the Lok Sabha, addresses both the
Houses, issues ordinances when they are not in session, and so on.
Statement 2 is correct: Though the President of India is not a member of either House of Parliament
and does not sit in Parliament to attend its meetings, he is an integral part of the Parliament.
Source: Laxmikanth chapter 22 – PARLIAMENT, page 479.

Q.9)
Ans) c
Exp) Option c is the correct answer.
Special Mention is a tool used to raise issues of public importance in Parliament.
Statement 1 is correct: The mechanism of ‘special mention’ is confined to Rajya Sabha. Its equivalent
procedural device in the Lok Sabha is known as ‘Notice (Mention) Under Rule 377’.
Statement 2 is correct: A matter which is not a point of order or which cannot be raised during
question hour, half-an hour discussion, short duration discussion, calling attention notice or under any
rule of the House can be raised under the special mention.
Source: Laxmikanth chapter 22 – PARLIAMENT, page 516.

Q.10)
Ans) b
Exp) Option b is the correct answer.
Statement 1 is correct: Union Budget 2023-24 has announced the setting up of the Urban
Infrastructure Development Fund (UIDF) through use of priority sector lending shortfall. Rs. 10,000
crore per annum will be made available for this purpose. The Fund will be managed by the National

Forum Learning Centre: Delhi - 2nd Floor, IAPL House, 19 Pusa Road, Karol Bagh, New Delhi - 110005 | Patna - 2nd floor, AG Palace, E Boring Canal
Road, Patna, Bihar 800001 | Hyderabad - 1st & 2nd Floor, SM Plaza, RTC X Rd, Indira Park Road, Jawahar Nagar, Hyderabad, Telangana 500020
9311740400, 9311740900 | https://academy.forumias.com | admissions@forumias.academy | helpdesk@forumias.academy
Page 5 of 45

PTS 2024 | B4 | L1 Test 2 - Solutions |

Housing Bank and will be used by public agencies to create urban infrastructure in Tier 2 and Tier 3
cities.
Statement 2 is incorrect: India has a Harmonized Master List of Infrastructure already in existence and
has been updated from time to time. The list is notified by the Ministry of Finance and includes sub-
sectors like transport and logistics, energy, water and sanitation etc. Inclusion in the list implies access
to concessional funds, promotion of projects and continuity of construction for the specified sub-
sectors. The Union Budget 2023-24 proposes to conduct a review of the List by an expert committee for
recommending the classification and financing framework suitable for ‘Amrit Kaal’.
Statement 3 is correct: Capital investment outlay has been proposed to be increased to 3.3% of GDP.
This has been increased for the third time in a row by 33%. It is expected to help in government’s efforts
to enhance growth potential and job creation, crowd-in private investments, and provide a cushion
against global headwinds.
Knowledge Base: National Housing Bank comes under Department of Financial Services, Ministry of
Finance.
Source: https://www.indiabudget.gov.in/doc/budget_speech.pdf

Q.11)
Ans) a
Exp) Option a is the correct answer.
A Financial Bill (I) is a bill that contains not only any or all the matters mentioned in Article 110, but also
other matters of general legislation. A financial bill (II) contains provisions involving expenditure from the
Consolidated Fund of India, but does not include any of the matters mentioned in Article 110.
Statement 1 is correct: In case of a disagreement between the two Houses over Financial bill I and
Financial bill II, the President can summon a joint sitting of the two Houses to resolve the deadlock.
Statement 2 is incorrect: A financial bill (I) can be introduced only in the Lok Sabha and not in the Rajya
Sabha, and it can be introduced only on the recommendation of the President. A financial bill (II) can be
introduced in either House of Parliament and recommendation of the President is not necessary for its
introduction.
Source: Laxmikanth chapter 22 – PARLIAMENT, page 525.

Q.12)
Ans) b
Exp) Option b is the correct answer.
Joint sitting is an extraordinary machinery provided by the Constitution to resolve a deadlock between
the two Houses over the passage of a bill.
Statement 1 is correct: The joint sitting is governed by the Rules of Procedure of Lok Sabha and not of
Rajya Sabha.
Statement 2 is incorrect: The Speaker of Lok Sabha presides over a joint sitting of the two Houses and
the Deputy Speaker, in his absence. If the Deputy Speaker is also absent from a joint sitting, the Deputy
Chairman of Rajya Sabha presides. If he is also absent, such other person as may be determined by the
members present at he joint sitting, presides over the meeting. The Chairman of Rajya Sabha does not
preside over a joint sitting as he is not a member of either House of Parliament.
Statement 3 is correct: The quorum to constitute a joint sitting is one-tenth of the total number of
members of the two Houses.
Statement 4 is incorrect: Every bill after being passed by both Houses of Parliament either
singly or at a joint sitting, is presented to the President for his assent. There are three alternatives
before the President:

Forum Learning Centre: Delhi - 2nd Floor, IAPL House, 19 Pusa Road, Karol Bagh, New Delhi - 110005 | Patna - 2nd floor, AG Palace, E Boring Canal
Road, Patna, Bihar 800001 | Hyderabad - 1st & 2nd Floor, SM Plaza, RTC X Rd, Indira Park Road, Jawahar Nagar, Hyderabad, Telangana 500020
9311740400, 9311740900 | https://academy.forumias.com | admissions@forumias.academy | helpdesk@forumias.academy
Page 6 of 45

PTS 2024 | B4 | L1 Test 2 - Solutions |


a. he may give his assent to the bill; or
b. he may withhold his assent to the bill; or
c. he may return the bill for reconsideration of the Houses.
If the President gives his assent to the bill, the bill becomes an act and is placed on the Statute Book.
Thus, President's assent is required in the case of a Joint Sitting.
Source: Laxmikanth chapter 22 – PARLIAMENT, page 527.

Q.13)
Ans) d
Exp) Option d is the correct answer.
The budget consists of two types of expenditure - the expenditure ‘charged’ upon the Consolidated Fund
of India and the expenditure ‘made’ from the Consolidated Fund of India. The charged expenditure is
non-votable by the Parliament, that is, it can only be discussed by the Parliament, while the other type
has to be voted by the Parliament. Some of the expenditure which are charged on the Consolidated Fund
are:
Option 1 is correct: The debt and liability of the Government of India for which it is liable to pay such
as interest, sinking fund charges and redemption charges and other expenditure relating to the raising
of loans and the service and redemption of debt are charged on the Consolidated Fund of India.
Option 2 is correct: Article 275 empowers the Parliament to make grants to the states which are in
need of financial assistance and not to every state. Also, different sums may be fixed for different states.
These sums are charged on the Consolidated Fund of India every year.
Option 3 is correct: The Central government can make loans to any state or give guarantees in respect
of loans raised by any state. Any sums required for the purpose of making such loans are to be charged
on the Consolidated Fund of India.
Option 4 is correct: Pensions of the judges of high courts are charged on the Consolidated Fund of
India. While the salaries and allowances of the judges, the salaries, allowances and pensions of the staff as
well as the administrative expenses of a high court are charged on the Consolidated Fund of the State.
Knowledge Base:
Some of the charged expenditure is as follows:
1) Emoluments and allowances of the President and other expenditure relating to his office.
2) Salaries and allowances of the Chairman and the Deputy Chairman of the Rajya Sabha and the
Speaker and the Deputy Speaker of the Lok Sabha.
3) Salaries, allowances and pensions of the judges of the Supreme Court.
4) Pensions of the judges of high courts.
5) Salary, allowances and pension of the Comptroller and Auditor General of India.
6) Salaries, allowances and pension of the chairman and members of the Union Public Service
Commission.
7) Administrative expenses of the Supreme Court, the office of the Comptroller and Auditor General of
India and the Union Public Service Commission including the salaries, allowances and pensions of the
persons serving in these offices.
8) The debt charges for which the Government of India is liable, including interest, sinking fund charges
and redemption charges and other expenditure relating to the raising of loans and the service and
redemption of debt.
9) Any sum required to satisfy any judgement, decree or award of any court or arbitral tribunal.
10) Any other expenditure declared by the Parliament to be so charged.
Source: Laxmikant, chapter 22: Parliament
Laxmikant, chapter 34: High Court
Lakshmikant Chapter 14: Center-State Relations

Forum Learning Centre: Delhi - 2nd Floor, IAPL House, 19 Pusa Road, Karol Bagh, New Delhi - 110005 | Patna - 2nd floor, AG Palace, E Boring Canal
Road, Patna, Bihar 800001 | Hyderabad - 1st & 2nd Floor, SM Plaza, RTC X Rd, Indira Park Road, Jawahar Nagar, Hyderabad, Telangana 500020
9311740400, 9311740900 | https://academy.forumias.com | admissions@forumias.academy | helpdesk@forumias.academy
Page 7 of 45

PTS 2024 | B4 | L1 Test 2 - Solutions |

Q.14)
Ans) d
Exp) Option d is the correct answer.
The General Budget has many demands. Each demand is voted separately by the Lok Sabha. During this
stage, the members of Parliament can discuss the details of the budget. They can also move motions to
reduce any demand for a grant. Such motions are called ‘cut motion’. Cut motion is of three kinds.
Pair 1 is incorrect: Token Cut Motion ventilates a specific grievance that is within the sphere of
responsibility of the Government of India. It states that the amount of demand should be reduced by Rs
100 (not Rs 1). Policy Cut Motion states that the amount of the demand be reduced to Re 1.
Pair 2 is incorrect: The economic cut motion calls for a reduction in the allocation of the demand to a
specific amount. It represents the economy that can be affected. Such a specified amount may either be
a lump-sum reduction in the demand or omission or reduction of an item in the demand. It does not
state that the amount of demand should be reduced by Rs 100.
Pair 3 is incorrect: Policy Cut Motion represents the disapproval of the policy underlying the demand. It
states that the amount of demand should be reduced to Rs 1. The members can also advocate an
alternative policy. It does not state that the amount of the demand be reduced by a specified amount
which may be lumpsum reduction in the demand.
Source: Laxmikant, chapter 22: Parliament

Q.15)
Ans) c
Exp) Option c is the correct answer.
The “Chip 4” or “Fab 4” alliance includes four of the world’s top producers of semiconductors: the U.S.,
Japan, Taiwan, and Korea. It represents more than 70 percent of the value of the global semiconductor
industry. It was first proposed by the US in March 2022 as part of wider plans aimed at enhancing the
"security" and "resilience" of semiconductor supply chains, including by reducing the world's reliance
on chips made in China. It is intended to cooperate on policy implementation that would support
sustainable semiconductor manufacturing in the member states’ home countries.
Source: https://www.reuters.com/technology/taiwan-says-fab-4-chip-group-held-first-senior-
officials-meeting-2023-02-25/

Q.16)
Ans) b
Exp) Option b is the correct answer.
The Thirteenth Amendment Act of 1962 granted the status of a state to Nagaland. This amendment
recognized the unique historical and cultural identity of the Naga Tribe and aimed to provide them with a
separate administrative and political entity within the Indian federal structure. By granting statehood to
Nagaland, the amendment acknowledged the aspirations and autonomy of the Naga people, promoting
their self-governance and development within the framework of the Indian Constitution.
Source: Laxmikanth, 6th Edition, Appendix IV Constitutional Amendments at a Glance

Q.17)
Ans) c
Exp) Option c is the correct answer.
The doctrine of ‘sovereignty of Parliament’ is associated with the British Parliament. There are no ‘legal’
restrictions on its authority and jurisdiction. The Indian Parliament, on the other hand, cannot be

Forum Learning Centre: Delhi - 2nd Floor, IAPL House, 19 Pusa Road, Karol Bagh, New Delhi - 110005 | Patna - 2nd floor, AG Palace, E Boring Canal
Road, Patna, Bihar 800001 | Hyderabad - 1st & 2nd Floor, SM Plaza, RTC X Rd, Indira Park Road, Jawahar Nagar, Hyderabad, Telangana 500020
9311740400, 9311740900 | https://academy.forumias.com | admissions@forumias.academy | helpdesk@forumias.academy
Page 8 of 45

PTS 2024 | B4 | L1 Test 2 - Solutions |

regarded as a sovereign body in the similar sense as there are ‘legal’ restrictions on its authority and
jurisdiction.
Option 1 is correct: Written Nature of the Constitution is a limit on the sovereignty of Indian
Parliament. The Constitution is the fundamental law of the land in our country. It has defined the
authority and jurisdiction of all the three organs of the Union government and the nature of inter-
relationship between them. Hence, Parliament has to operate within the limits prescribed by the
Constitution.
Option 2 is correct: Federal System of Government limits the sovereignty of Indian Parliament. India
has a federal system of government with a constitutional division of powers between the Union and the
states. Both have to operate within the spheres allotted to them. Hence, the lawmaking authority of the
Parliament gets confined to the subjects enumerated in the Union List and Concurrent List and does
not extend to the subjects enumerated in the State List except during some exceptional circumstances
and that too for a short period. Britain, on the other hand, has a unitary system of government and
hence, all the powers are vested in the Centre.
Option 3 is incorrect: The Emergency provisions enable the Central government to meet any abnormal
situation effectively. The Central government becomes all powerful and the states go into the total
control of the Centre. It converts the federal structure into a unitary one without a formal amendment
of the Constitution. This kind of transformation of the political system from federal during normal times
to unitary during Emergency is a unique feature of the Indian Constitution. Hence, the Emergency
provisions do not limit the sovereignty of Indian Parliament, rather make it more powerful.
Option 4 is correct: System of Judicial Review limits the sovereignty of Indian Parliament.
The adoption of an independent Judiciary with the power of judicial review also restricts the
supremacy of our Parliament. Both the Supreme Court and high courts can declare the laws enacted by
the Parliament as void and ultra vires (unconstitutional), if they contravene any provision of the
Constitution. On the other hand, there is no system of judicial review in Britain.
Source: Laxmikant, chapter 22: Parliament

Q.18)
Ans) b
Exp) Option b is the correct answer.
The Public Accounts Committee is a committee of selected Members of Parliament, constituted by the
Parliament of India, for the purpose of auditing the revenue and the expenditure of the Government of
India. At present, it consists of 22 members (15 from the Lok Sabha and 7 from the Rajya Sabha). The
members are elected by the Parliament every year from amongst its members according to the principle
of proportional representation by means of the single transferable vote.
Statement 1 is correct: Public Accounts Committee was first set up in the wake of the Montague-
Chelmsford Reforms. It was set up first in 1921 under the provisions of the Government of India Act of
1919 and has since been in existence.
Statement 2 is correct: Public Accounts Committee can examine the accounts of autonomous and
semiautonomous bodies if audit of these bodies is conducted by the CAG. It also examines the accounts
of state corporations, trading concerns and manufacturing projects and the audit report of CAG on them
(except those public undertakings which are allotted to the Committee on Public Undertakings)
Statement 3 is incorrect: The effectiveness of the role of the committee is limited by the following
factors:
1) It is not concerned with the questions of policy in broader sense.
2) It conducts a post-mortem examination of accounts (showing the expenditure already incurred).
3) It cannot intervene in the matters of day-to-day administration.

Forum Learning Centre: Delhi - 2nd Floor, IAPL House, 19 Pusa Road, Karol Bagh, New Delhi - 110005 | Patna - 2nd floor, AG Palace, E Boring Canal
Road, Patna, Bihar 800001 | Hyderabad - 1st & 2nd Floor, SM Plaza, RTC X Rd, Indira Park Road, Jawahar Nagar, Hyderabad, Telangana 500020
9311740400, 9311740900 | https://academy.forumias.com | admissions@forumias.academy | helpdesk@forumias.academy
Page 9 of 45

PTS 2024 | B4 | L1 Test 2 - Solutions |

4) It is not vested with the power of disallowance of expenditures by the departments.


5) It is not an executive body and hence, cannot issue an order.
6) Only the Parliament can take a final decision on its findings.
Source: Laxmikant, chapter 22: Parliamentary Committees

Q.19)
Ans) b
Exp) Option b is the correct answer.
Statement 1 is incorrect: Committee on Subordinate Legislation is not a departmental standing
committee. It is one of the committees established to scrutinize and control the functioning of the
government. It was constituted in 1953.
Statement 2 is correct: Committee on Subordinate Legislation examines and reports to the House
whether the powers to make regulations, rules, sub-rules and bye-laws delegated by the Parliament or
conferred by the Constitution to the Executive are being properly exercised by it.
Statement 3 is correct: In both the Houses of the Parliament, the committee consists of 15 members.
Source: M Laxmikant chapter 23, 5th edition

Q.20)
Ans) a
Exp) Option a is the correct answer.
Recently, it is reported that a rapidly spreading disease, commonly known as yellow band disease, is
killing corals over vast stretches of the sea floor of Thailand. Scientists believe overfishing, pollution and
rising water temperatures because of climate change might be making the reefs more vulnerable to
yellow-band disease.
Statement 1 is correct: Scientists believe overfishing, pollution and rising water temperatures because of
climate change may be making the reefs more vulnerable to yellow-band disease. The impact of the
yellow band disease cannot be reversed, unlike the effects of coral bleaching.
Statement 2 is incorrect: It was first spotted decades ago and has caused widespread damage to the
reefs in the Caribbean (thus, first occurrence is not from Australia). Over 600 acres of the sea is
reported to have been infected due to this disease.
Source: https://oceanservice.noaa.gov/education/tutorial_corals/media/supp_coral10b.html
https://www.thehindu.com/sci-tech/energy-and-environment/watch-why-are-corals-in-thailand-
getting-destroyed/article66396891.ece
https://www.hindustantimes.com/world-news/it-just-dies-what-is-the-yellow-band-disease-
ravaging-thailand-s-coral-reefs-101672205686483.html

Q.21)
Ans) c
Exp) Option c is the correct answer.
Article 368 in Part XX of the Constitution deals with the powers of Parliament to amend the Constitution
and its procedure. It states that the Parliament may amend the Constitution by way of addition, variation
or repeal any provision in accordance with the procedure laid down for the purpose (except the Basic
Structure of the Constitution). 105 amendments have been made to the constitution till date.
Statement 1 is correct. The 7th Constitutional Amendment Act,1956 has extended the jurisdiction of High
Courts to Union Territories. It also provided for the establishment of a common High Court for two or
more States.

Forum Learning Centre: Delhi - 2nd Floor, IAPL House, 19 Pusa Road, Karol Bagh, New Delhi - 110005 | Patna - 2nd floor, AG Palace, E Boring Canal
Road, Patna, Bihar 800001 | Hyderabad - 1st & 2nd Floor, SM Plaza, RTC X Rd, Indira Park Road, Jawahar Nagar, Hyderabad, Telangana 500020
9311740400, 9311740900 | https://academy.forumias.com | admissions@forumias.academy | helpdesk@forumias.academy
Page 10 of 45

PTS 2024 | B4 | L1 Test 2 - Solutions |

Statement 2 is correct. The 9th Constitutional Amendment Act, 1960 facilitated the cession of Indian
territory of Berubari Union (located in West Bengal) to Pakistan as provided in the Indo-Pakistan
Agreement
Source: Appendix IV Constitutional Amendments at a Glance

Q.22)
Ans) b
Exp) Option b is the correct answer.
Privileges of a state legislature are a sum of special rights, immunities and exemptions enjoyed by the
Houses of state legislature, their committees and their members. The privileges of a state legislature can
be classified into two broad categories—
1) collective privileges- Those that are enjoyed by each House of the state legislature collectively, and
2) individual privileges- Those that are enjoyed by the members individually.
Some of the collective privileges are:
1) It has the right to publish its reports, debates and proceedings and also the right to prohibit others
from publishing the same. Hence statement 1 is correct.
2) It can exclude strangers from its proceedings and hold secret sittings to discuss some important
matters.
3) It can make rules to regulate its own procedure and the conduct of its business and to adjudicate upon
such matters. Hence statement 2 is correct.
4) The courts are prohibited to inquire into the proceedings of a House or its Committees.
Exemption for members of state legislature from jury service is an example of individual privilege and
not collective privilege. Using this privilege, members can refuse to give evidence and appear as a witness
in a case pending in a court when the state legislature is in session. Hence statement 3 is incorrect.
Source: Chapter 33 M Laxmikant 5th edition

Q.23)
Ans) c
Exp) Option c is the correct answer.
Every bill, after it is passed by the assembly or by both the Houses in case of a bicameral legislature, is
presented to the governor for his assent. When a bill is reserved by the Governor for the consideration of
the President, there are following possible actions that can be taken by the President:
1) The President may either give his assent to the bill, or (Hence, Option 1 is correct)
2) Withhold his assent to the bill, or (Hence, Option 2 is correct)
3) Return the bill for reconsideration of the House or Houses of the state legislature. (Hence, Option 3 is
correct)
There is no provision that President transfers the bill to the Parlaiment if Governor of the state have
reserved the bill for the consideration of President. (Hence, Option 4 is incorrect)
Knowledge Base: When a bill is returned, the House or Houses have to reconsider it within a period of six
months. The bill is presented again to the presidential assent after it is passed by the House or Houses
with or without amendments. It is not mentioned in the Constitution whether it is obligatory on the part
of the president to give his assent to such a bill or not.
Also, the time limit within which the President has to consider the Bill reserved by the Governor is not
mentioned in the Constitution.
Source: Chapter 33 M Laxmikant 5th edition

Forum Learning Centre: Delhi - 2nd Floor, IAPL House, 19 Pusa Road, Karol Bagh, New Delhi - 110005 | Patna - 2nd floor, AG Palace, E Boring Canal
Road, Patna, Bihar 800001 | Hyderabad - 1st & 2nd Floor, SM Plaza, RTC X Rd, Indira Park Road, Jawahar Nagar, Hyderabad, Telangana 500020
9311740400, 9311740900 | https://academy.forumias.com | admissions@forumias.academy | helpdesk@forumias.academy
Page 11 of 45

PTS 2024 | B4 | L1 Test 2 - Solutions |

Q.24)
Ans) c
Exp) Option c is the correct answer.
Calling Attention Motion is introduced in the Parliament by a member to call the attention of a minister
to a matter of urgent public importance, and to seek an authoritative statement from him on that
matter. Like the zero hour, it is also an Indian innovation in the parliamentary procedure and has been in
existence since 1954. However, unlike the zero hour, it is mentioned in the Rules of Procedure.
Option a is incorrect: A member can raise a point of order when the proceedings of the House do not
follow the normal rules of procedure. A point of order should relate to the interpretation or
enforcement of the Rules of the House or such articles of the Constitution that regulate the business of
the House and should raise a question that is within the cognizance of the Speaker. It is usually raised by
an opposition member in order to control the government. It is an extraordinary device as it suspends
the proceedings before the House. No debate is allowed on a point of order.
Option b is incorrect: The first hour of every parliamentary sitting is slotted for Question Hour. During
this time, the members ask questions and the ministers usually give answers. The
questions are of three kinds, namely, starred, unstarred and short notice.
Option d is incorrect: Unlike the question hour, the zero hour is not mentioned in the Rules of
Procedure. Thus, it is an informal device available to the members of the Parliament to raise matters
without any prior notice. The zero hour starts immediately after the question hour and lasts until the
agenda for the day (i.e., regular business of the House) is taken up.
Source: Laxmikanth Chapter 22 Parliament

Q.25)
Ans) c
Exp) Option c is the correct answer.
Recently, the Kamioka Liquid Scintillator Antineutrino Detector (KamLAND) experiment in Japan has
found no evidence that neutrinos are their own antiparticles.
The Kamioka Liquid Scintillator Antineutrino Detector (KamLAND) is a neutrino and antineutrino
detector located in the Kamioka Observatory in Japan. It is designed to study the properties of
neutrinos and antineutrinos, which are subatomic particles that have very little mass and interact very
weakly with matter. KamLAND is an important tool for studying neutrinos and antineutrinos because it
can detect neutrinos from various sources, including the sun, nuclear reactors, and the Earth's mantle.
Knowledge Base:
Neutrino: A neutrino is a subatomic particle that is very similar to an electron, but has no electrical
charge and a very small mass.
Antiparticle: In quantum theory, every type of particle is associated with an antiparticle with the same
mass but with opposite physical charges. For example, the antiparticle of the electron is the positron.
Source) https://www.thehindu.com/sci-tech/science/explained-are-neutrinos-their-own-
antiparticles/article66532648.ece

Q.26)
Ans) a
Exp) Option a is the correct answer.
A money bill in India is a type of legislation that pertains exclusively to matters related to taxation,
borrowing of money by the government, or expenditure from consolidated funds. It can only be
introduced in the lower house of the Parliament (Lok Sabha) or the Legislative Assembly (in the case of

Forum Learning Centre: Delhi - 2nd Floor, IAPL House, 19 Pusa Road, Karol Bagh, New Delhi - 110005 | Patna - 2nd floor, AG Palace, E Boring Canal
Road, Patna, Bihar 800001 | Hyderabad - 1st & 2nd Floor, SM Plaza, RTC X Rd, Indira Park Road, Jawahar Nagar, Hyderabad, Telangana 500020
9311740400, 9311740900 | https://academy.forumias.com | admissions@forumias.academy | helpdesk@forumias.academy
Page 12 of 45

PTS 2024 | B4 | L1 Test 2 - Solutions |


state legislatures) and not in the upper house (Rajya Sabha or Legislative Council). Money bills play a
crucial role in shaping the fiscal policies and financial matters of the country or state.
Statement 1 is correct- Every money bill is considered to be a government bill and can be introduced
only by a minister. This ensures that matters concerning taxation, government finances, and expenditure
are under the direct control of the executive branch. The requirement for a minister to introduce a
money bill emphasizes the government's responsibility and accountability in matters related to public
funds. This procedure helps maintain financial discipline and transparency in the legislative process.
Statement 2 is incorrect- When a Money Bill is presented to the governor, he can either give his assent,
or withhold his assent, or reserve the bill for presidential assent. However, the governor cannot return
the bill for reconsideration by the state legislature. Typically, the governor gives their assent to a money
bill since it is introduced in the state legislature with their prior permission.
When a money bill is presented to the President, he can either give his assent to the bill or withhold his
assent to the bill. However, the President cannot return the bill for reconsideration of the parliament.
Typically, the President gives their assent to a money bill since it is introduced in the Parliament with
their prior permission.
Statement 3 is incorrect- In case of Money bill, the constitution does not offer a mechanism to resolve
deadlocks between the two Houses i.e. Lok Sabha and Rajya Sabha. In such cases, the will of the Lok
Sabha takes precedence over that of the Rajya Sabha if they disagree on a Money bill passed by the
former.
In the case of the State Legislature as well, the constitution does not offer a mechanism to resolve
deadlocks between the two Houses regarding Money Bills. In such cases, the will of the Legislative
Assembly takes precedence over that of the Legislative Council if they disagree on a Money bill passed by
the former.
Source: Laxmikanth, 6th Edition, Chapters 22 and 33

Q.27)
Ans) a
Exp) Option a is the correct answer.
State legislatures have the authority to determine the languages to be used as the floor languages for
conducting business within their respective assemblies. The choice of languages may vary from state to
state, depending on linguistic diversity and regional preferences.
Statement 1 is correct- The Constitution designates the official language(s) of the state or either Hindi or
English as the languages for conducting business in the state legislature. This provision ensures that
legislative proceedings, including discussions and transactions, are carried out in the prescribed official
language(s), facilitating effective communication and adherence to constitutional guidelines within the
state legislature.
Statement 2 is incorrect- The presiding officer, and not the governor has the authority to allow a
member to address the House in their native language. This provision recognizes the importance of
linguistic diversity and facilitates effective communication for members who are more comfortable
expressing themselves in their mother tongue.
Statement 3 is incorrect- The state legislature (and not the Governor) has the power to determine the
continuation or discontinuation of English as a floor language after fifteen years from the Constitution's
commencement. However, in the cases of Himachal Pradesh, Manipur, Meghalaya, and Tripura, the time
limit is extended to twenty-five years, while Arunachal Pradesh, Goa, and Mizoram have a time limit of
forty years.
Source: Laxmikanth, 6th Edition- Chapter 33

Forum Learning Centre: Delhi - 2nd Floor, IAPL House, 19 Pusa Road, Karol Bagh, New Delhi - 110005 | Patna - 2nd floor, AG Palace, E Boring Canal
Road, Patna, Bihar 800001 | Hyderabad - 1st & 2nd Floor, SM Plaza, RTC X Rd, Indira Park Road, Jawahar Nagar, Hyderabad, Telangana 500020
9311740400, 9311740900 | https://academy.forumias.com | admissions@forumias.academy | helpdesk@forumias.academy
Page 13 of 45

PTS 2024 | B4 | L1 Test 2 - Solutions |

Q.28)
Ans) d
Exp) Option d is the correct answer.
Double Membership of the Parliament or State Legislature is a provision that prohibits a member to have
multiple seats in the Parliament or State Legislature.
Statement 1 is correct- An individual cannot simultaneously be a member of both Parliament and a state
legislature. If someone is elected to both, their seat in Parliament becomes vacant unless they resign from
the state legislature within 14 days.
Statement 2 is incorrect- Upon being elected to both Houses of Parliament, an individual must promptly
declare their preferred House within 10 days. Failure to do so will result in the vacancy of their seat in the
Rajya Sabha.
Statement 3 is correct- When a sitting member of one House is elected to the other House, their seat in
the first House becomes vacant, ensuring that they can only hold membership in one House at a time and
preventing dual representation. This helps maintain the separation of roles and responsibilities between
the two Houses of Parliament.
Statement 4 is correct- In order to avoid holding multiple seats in a House, when a person is elected to
two seats, it is essential for them to exercise their choice and select one seat to retain. Failing to make a
decision within the stipulated time period results in both seats becoming vacant, ensuring adherence to
the principle of single membership and preventing any potential conflicts or duplication of
representation.
Source: Laxmikanth, 6th edition, Chapter 33

Q.29)
Ans) a
Exp) Option a is the correct answer.
The legislative council, also known as the upper house, is a vital component of the state legislature in
India. It provides a platform for thoughtful debates, discussions, and legislative scrutiny. The council plays
a crucial role in ensuring balanced representation and deliberation in the legislative process.
Statement 1 is correct- Similar to the Rajya Sabha, the legislative council is a perpetual chamber,
meaning it is a permanent body that does not dissolve. However, one-third of its members retire every
two years, allowing members to serve a term of six years. This system ensures a regular turnover of
members while maintaining the council's continuity.
Statement 2 is incorrect- Like the Rajya Sabha, the legislative council is a continuing chamber, that is, it
is a permanent body and is not subject to dissolution. The members of Legislative council have a tenure of
6 years.
Statement 3 is incorrect- The members who retire from the legislative council remain eligible for re-
election and re-nomination without any limit on the number of terms they can serve. This provision
allows experienced and knowledgeable individuals to continue their valuable contributions to the council,
fostering continuity and expertise in legislative proceedings.
Source: Laxmikanth, 6th edition, chapter 33

Q.30)
Ans) c
Exp) Option c is the correct answer.
Geomagnetic storms are disturbances in the Earth's magnetic field that are caused by the interaction
between the Earth's magnetosphere and solar wind. The solar wind is a stream of charged particles that
flows from the Sun and interacts with the Earth's magnetic field. When the solar wind encounters the

Forum Learning Centre: Delhi - 2nd Floor, IAPL House, 19 Pusa Road, Karol Bagh, New Delhi - 110005 | Patna - 2nd floor, AG Palace, E Boring Canal
Road, Patna, Bihar 800001 | Hyderabad - 1st & 2nd Floor, SM Plaza, RTC X Rd, Indira Park Road, Jawahar Nagar, Hyderabad, Telangana 500020
9311740400, 9311740900 | https://academy.forumias.com | admissions@forumias.academy | helpdesk@forumias.academy
Page 14 of 45

PTS 2024 | B4 | L1 Test 2 - Solutions |


Earth's magnetosphere, it can cause fluctuations and disturbances in the magnetic field, which can lead
to a geomagnetic storm. The consequences of ‘Geomagnetic storm’ in the Earth's magnetic field are:
Option 1 is correct: Global navigation systems rely on precise measurements of the Earth's magnetic field
to accurately determine location and orientation. Geomagnetic storms can disrupt these systems by
causing fluctuations in the magnetic field, leading to errors in navigation and communication.
Option 2 is correct: Geomagnetic storms can cause charged particles from the Sun to interact with the
Earth's magnetic field, resulting in colorful displays of light known as auroras. These are a natural and
beautiful phenomenon that can be observed in regions near the Earth's poles.
Option 3 is correct: Geomagnetic storms can cause damage to satellite electronics by inducing
electrical currents that can overload circuits and damage sensitive equipment. This can lead to
communication disruptions, loss of data, and other issues that can impact satellite-based services and
infrastructure.
Option 4 is incorrect: Oceanic gyre circulation is driven by a combination of factors, including the Earth's
rotation, prevailing winds, and ocean currents. Geomagnetic storms are not known to cause a reversal
of oceanic gyre circulation, which is a much larger and more complex process.
Source: https://tech.hindustantimes.com/tech/news/geomagnetic-storm-hits-earth-solar-wind-
sparks-red-auroras-in-us-71676635258970.html

Q.31)
Ans) c
Exp) Option c is the correct answer.
The office of the president is the highest constitutional position in India. The president serves as the head
of state, representing the unity and sovereignty of the nation. The president's role includes various
functions such as appointing the prime minister, assenting to legislation, and safeguarding the
Constitution.
Statement 1 is incorrect- The president cannot be a member of either the Parliament or a state
legislature. If a person holding a seat in either House is elected as the president, they are considered to
have resigned from that position upon assuming the office of the president.
Statement 2 is correct- In addition to the restrictions on being a member of Parliament or a state
legislature, the president is also prohibited from holding any other office of profit. This requirement
ensures that the president can impartially fulfill their duties without being influenced by any conflicting
interests or engagements. It helps maintain the integrity and independence of the office of the president.
Statement 3 is correct- The president is entitled to the use of their official residence without having to
pay rent. This provision ensures that the president has a suitable and secure place of residence while
performing their duties. It also upholds the dignity and prestige associated with the presidential office,
providing the necessary amenities and facilities for the president's residence.
Statement 4 is correct- During the president's term of office, their emoluments and allowances cannot
be reduced or diminished. This provision safeguards the president's financial interests and ensures
stability in their remuneration, promoting the independence and dignity of the presidential office. It
prevents any potential manipulation or interference with the president's compensation during their
tenure.
Source: Laxmikanth, 6th edition, chapter 17

Q.32)
Ans) a
Exp) Option a is the correct answer.

Forum Learning Centre: Delhi - 2nd Floor, IAPL House, 19 Pusa Road, Karol Bagh, New Delhi - 110005 | Patna - 2nd floor, AG Palace, E Boring Canal
Road, Patna, Bihar 800001 | Hyderabad - 1st & 2nd Floor, SM Plaza, RTC X Rd, Indira Park Road, Jawahar Nagar, Hyderabad, Telangana 500020
9311740400, 9311740900 | https://academy.forumias.com | admissions@forumias.academy | helpdesk@forumias.academy
Page 15 of 45

PTS 2024 | B4 | L1 Test 2 - Solutions |

Articles 153 to 167 in Part VI of the Constitution deal with the state executive. The state executive
consists of the governor, the chief minister, the council of ministers and the advocate general of the
state.
Statement 1 is correct: The governor is the chief executive head of the state. But, like the President, he is
a nominal executive head (titular or constitutional head). He is appointed by the President by warrant
under his hand and seal.
Statement 2 is incorrect: The governor is neither directly elected by the people nor indirectly elected by
a specially constituted electoral college as is the case with the president. Supreme Court in 1979 held that
the office of governor of a state is not an employment under the Central government. It is an
independent constitutional office and is not under the control of or subordinate to the Central
government.
Source: laxmikanth, chapter 30

Q.33)
Ans) d
Exp) Option d is the correct answer.
The Constitution of India empowers the President with various powers. Some of which are explained
below:
Statement 1 is correct. Money bills can be introduced in the Parliament only with the prior
recommendation of the President of India.
Statement 2 is correct. A demand for a grant cannot be made without the recommendation of the
President of India.
Statement 3 is correct. To meet any unforeseen expenditures, the President can make advances out of
the Contingency Fund of India.
Statement 4 is correct. The President can summon or prorogue the Parliament and dissolve the Lok
Sabha. This is the Legislative power of President.
Source: M. Laxmikanth Indian Polity, Chapter-17, Page 410

Q.34)
Ans) b
Exp) Option b is the correct answer.
The President and the Governors of the States of India were provided with the pardoning power under
Articles 72 and 161 of the Constitution, respectively. This power has been provided to avoid injustice from
unjust laws or from judgements that resulted in injustice.
Statement 1 is correct. Constitution of India under Article 72 has empowered the President of India to
pardon, reprieve, respite, remit, suspend or commute the punishment or sentence of any person
convicted of any offence against a Central law. The Governor of a state under Article 161 can pardon
the punishment or sentence of any person convicted of any offence against a State law.
Statement 2 is incorrect. The President can pardon, reprieve, respite, remit, suspend or commute a
death sentence while the Governor of a state cannot pardon a death sentence. Only the President has
the power to pardon a death sentence. Even if a state law prescribes the death penalty, it is the
President, not the governor, who has the power to grant the death sentence. However, a death sentence
may be suspended, remitted, or commuted by the Governor of a state.
Statement 3 is correct. The President can grant pardon, reprieve, respite, suspend, remit or commute
the punishment or sentence by a court-martial (military court) while the Governor of a State has no say
in such matters.

Forum Learning Centre: Delhi - 2nd Floor, IAPL House, 19 Pusa Road, Karol Bagh, New Delhi - 110005 | Patna - 2nd floor, AG Palace, E Boring Canal
Road, Patna, Bihar 800001 | Hyderabad - 1st & 2nd Floor, SM Plaza, RTC X Rd, Indira Park Road, Jawahar Nagar, Hyderabad, Telangana 500020
9311740400, 9311740900 | https://academy.forumias.com | admissions@forumias.academy | helpdesk@forumias.academy
Page 16 of 45

PTS 2024 | B4 | L1 Test 2 - Solutions |

Source: M. Laxmikanth Indian Polity Sixth Edition, Chapter-17 (Page 685)

Q.35)
Exp) Option b is the correct answer.
Services Trade Restrictiveness Index (STRI) was launched in 2014 by Organization for Economic Co-
operation and Development (OECD) and provides information on regulations affecting trade in services
in 22 sectors across all member countries.
The regulatory database and indices are updated annually in December. There is also a Digital STRI that
identifies, catalogues, and quantifies cross-cutting barriers that affect services traded digitally.
Scoring: This index takes values between zero and one, one representing a totally closed and zeros
representing a fully open sector.
In the recent services trade restrictiveness index (STRI), India’s rank improved a notch to 47 from 48
among 50 countries surveyed by the Organization for Economic Co-operation and Development
(OECD).
Source: https://www.business-standard.com/article/economy-policy/india-s-rank-improves-in-oecd-
services-trade-restrictiveness-index-123021601063_1.html

Q.36)
Ans) c
Exp) Option c is the correct answer.
Statement 1 is correct. The Constitution of India under Article 78 states that it shall be the duty of the
Prime Minister of India to communicate all the decisions taken by the Council of Ministers with
regards to the proposals for legislation and the administration of the Union’s affairs.
Statement 2 is correct. If the President of India requires that the Prime Minister shall submit for the
consideration of the council of ministers any matter on which a decision has been taken by a minister,
but which has not been considered by the council.
Statement 3 is correct. The Prime Minister of India advises the President in the matters of the
appointment of important officials like Attorney General of India, Comptroller and Auditor General of
India, UPSC chairman and members, Election Commissioners, Chairman and members of the Finance
Commission, etc.
Source: M. Laxmikanth Indian Polity, Chapter-19

Forum Learning Centre: Delhi - 2nd Floor, IAPL House, 19 Pusa Road, Karol Bagh, New Delhi - 110005 | Patna - 2nd floor, AG Palace, E Boring Canal
Road, Patna, Bihar 800001 | Hyderabad - 1st & 2nd Floor, SM Plaza, RTC X Rd, Indira Park Road, Jawahar Nagar, Hyderabad, Telangana 500020
9311740400, 9311740900 | https://academy.forumias.com | admissions@forumias.academy | helpdesk@forumias.academy
Page 17 of 45

PTS 2024 | B4 | L1 Test 2 - Solutions |

Q.37)
Ans) b
Exp) Option b is the correct answer.
The words ‘council of ministers’ and ‘cabinet’ are often used interchangeably though there is a definite
distinction between them. They differ from each other in respects of composition, functions, and role.
Statement 1 is correct. The Council of Ministers is a wider body consisting of 60 to 70 ministers and
includes all the three categories of ministers, that is, cabinet ministers, ministers of state, and deputy
ministers. The Cabinet is a smaller body consisting of 15 to 20 ministers which forms a part of the
council of ministers.
Statement 2 is correct. The Council of Ministers does not meet as a body to transact government
business and has no collective functions while the Cabinet frequently meets as a body to deliberate and
take decisions regarding the transaction of government business.
Statement 3 is incorrect. The Cabinet directs the council of ministers by taking policy decisions which
are binding on all ministers. Thus, it can be said that the functions of the Council of Ministers are
determined by the Cabinet.
Source: M. Laxmikanth Indian Polity, Chapter-20

Q.38)
Ans) a
Exp) Option a is the correct answer.
The cabinet is the highest decision-making body in the official sense, consisting of the prime minister as
its head and 15 to 20 most senior ministers. However, a much smaller body known as the 'Inner Cabinet'
or 'Kitchen Cabinet' has emerged as the true seat of power.
Statement 1 is correct. Kitchen Cabinet is an informal body (extra-constitutional body) that is not
mentioned in the Constitution of India. It consists of the Prime Minister and two to four influential
colleagues in whom he has faith and with whom he can discuss every problem. It advises the prime
minister on important political and administrative issues and assists him in making crucial decisions.
Statement 2 is incorrect. The Kitchen Cabinet is composed of not only cabinet ministers but also
outsiders like friends and family members of the Prime Minister.
Statement 3 is incorrect. It comprises of the Prime Minister, other cabinet members, and those from
outside the government in whom the Prime Minister has confidence and trust. The advice tendered by
the Kitchen Cabinet is non-binding in nature. It is also found in the United States and the United
Kingdom.
Source: Laxmikant chapter 20

Q.39)
Ans) a
Exp) Option a is the correct answer.
The Governor is the chief executive head of the state. Articles 153 to 167 in Part VI of the Constitution
deal with the state executive.
Option 1 is correct. The Constitution lays down only two qualifications for the appointment of a person
as a governor. These are:
1. He should be a citizen of India.
2. He should have completed the age of 35 years.
Option 2 is incorrect. Two conventions have also developed in this regard over the years.
1. He should be an outsider, that is, he should not belong to the state where he is appointed so that he is
free from the local politics.

Forum Learning Centre: Delhi - 2nd Floor, IAPL House, 19 Pusa Road, Karol Bagh, New Delhi - 110005 | Patna - 2nd floor, AG Palace, E Boring Canal
Road, Patna, Bihar 800001 | Hyderabad - 1st & 2nd Floor, SM Plaza, RTC X Rd, Indira Park Road, Jawahar Nagar, Hyderabad, Telangana 500020
9311740400, 9311740900 | https://academy.forumias.com | admissions@forumias.academy | helpdesk@forumias.academy
Page 18 of 45

PTS 2024 | B4 | L1 Test 2 - Solutions |

2. While appointing the governor, the president is required to consult the chief minister of the state
concerned, so that the smooth functioning of the constitutional machinery in the state is ensured.
However, both conventions have been violated in some of the cases.
Option 3 is incorrect. 'He should be qualified for election as a member of the Lok Sabha'- This
qualification is for a person contesting for election as the President, and not for the Governor.
Source: laxmikant chapter 30

Q.40)
Ans) d
Exp) Option d is the correct answer.
Option d is correct: A Financial contagion is the spread of an economic crisis from one market or
region to another and can occur at both a domestic or international level. It often causes a domino
effect on financial markets, financial institutions, and the broader economy.
1) Financial contagion can occur due to various factors, including the interconnectedness of financial
markets, the globalization of financial transactions, the proliferation of complex financial
instruments, and the increased speed and volume of financial transactions.
2) This may lead to a loss of confidence in the financial system and a rush to exit investments or liquidate
assets.
3) The 1997 Asian financial markets crisis and the 2008 global financial crisis are the prime example of
financial contagion.
Source: https://www.investopedia.com/terms/c/contagion.asp
https://world101.cfr.org/global-era-issues/monetary-policy-and-currencies/global-consequences-
financial-contagion
https://www.oecd.org/economy/monetary/50556019.pdf

Q.41)
Ans) c
Exp) Option c is the correct answer.
The Governor is both a titular or constitutional leader and an agent of the Centre. The Governor, like the
President, has a number of privileges and immunities.
Statement 1 is correct. The Governor is not completely immune from Civil proceedings for his personal
acts. The Civil proceedings can be instituted against the Governor during his term of office after giving
two months’ notice, in respect of his personal acts.
Statement 2 is correct. He is personally immune from legal liability for his official actions. During his
tenure of office, he is exempt from criminal proceedings, including for personal activities. He cannot be
arrested or imprisoned. The governor cannot be summoned for interrogation under constitutional
immunity granted to governors under Article 361(2).
Statement 3 is correct. When a governor is responsible for two or more states, the emoluments and
allowances payable to him are shared by the states in such proportion as the President may determine.
The emoluments and allowances of the Governor shall not be diminished during his term of office.
Source: laxmikant chapter 30

Q.42)
Ans) a
Exp) Option a is the correct answer.

Forum Learning Centre: Delhi - 2nd Floor, IAPL House, 19 Pusa Road, Karol Bagh, New Delhi - 110005 | Patna - 2nd floor, AG Palace, E Boring Canal
Road, Patna, Bihar 800001 | Hyderabad - 1st & 2nd Floor, SM Plaza, RTC X Rd, Indira Park Road, Jawahar Nagar, Hyderabad, Telangana 500020
9311740400, 9311740900 | https://academy.forumias.com | admissions@forumias.academy | helpdesk@forumias.academy
Page 19 of 45

PTS 2024 | B4 | L1 Test 2 - Solutions |

Option a is correct. A Governor is appointed for a five-year term commencing on the day he enters
office. This five-year tenure, however, is subject to the President's pleasure. The Constitution does not
specify any grounds for the President to remove a governor.
The President has the authority to transfer a Governor appointed to one state to another for the
remainder of the term. Furthermore, a Governor whose term has expired may be reappointed in the
same or another state.
Source: laxmikant chapter 30

Q.43)
Ans) c
Exp) Option c is the correct answer.
The Governor is the chief executive head of the State. But like the President, he is the nominal executive
head. The Governor also acts as an agent of the Central Government. A Governor possesses executive,
legislative, financial and judicial powers more or less analogous to the President of India.
Statement 1 is correct. All executive actions of the government of a state are formally taken in the
Governor's name. He appoints the Chief Minister and other ministers. They also hold office during his
pleasure.
Statement 2 is incorrect. He appoints the chairman and members of the State Public Service
Commission. However, they can be removed only by the president and not by a Governor.
Statement 3 is correct. He appoints the state election commissioner and determines his conditions of
service and tenure of office. However, the state election commissioner can be removed only in like
manner and on the like grounds as a judge of a High Court.
Statement 4 is correct. He appoints the advocate general of a state and determines his remuneration.
The advocate general holds office during the pleasure of the Governor.
Source: laxmikant chapter 30.
https://www.legalserviceindia.com/legal/article-2732-constitutional-position-of-the-president-and-
governor-relation-with-the-council-of-ministers-.html

Q.44)
Ans) b
Exp) Option b is the correct answer.
According to theory of collective ministerial responsibility, the Ministers are responsible to the House of
the People. It is a business of the House of the People to see that the Cabinet remains in power so long as
it retains the confidence of the majority in that House. It is explicitly expressed in Art. 75 (3) of our
Constitution. Art. 75 (3) reads “The Council of Ministers shall be collectively responsible to the House of
the People”.
The principle of collective responsibility also means that the Cabinet decisions bind all cabinet ministers
even if they differed in the cabinet meeting. It is the duty of every minister to stand by cabinet decisions
and support them both within and outside the Parliament.
Source: DD BASU CH. 12
https://indiankanoon.org/doc/1336375/

Q.45)
Ans) b
Exp) Option b is the correct answer.
The RAMP scheme aims at improving access of MSMEs to market and credit, technology upgradation
and addressing issues of delayed payments and greening of MSMEs.

Forum Learning Centre: Delhi - 2nd Floor, IAPL House, 19 Pusa Road, Karol Bagh, New Delhi - 110005 | Patna - 2nd floor, AG Palace, E Boring Canal
Road, Patna, Bihar 800001 | Hyderabad - 1st & 2nd Floor, SM Plaza, RTC X Rd, Indira Park Road, Jawahar Nagar, Hyderabad, Telangana 500020
9311740400, 9311740900 | https://academy.forumias.com | admissions@forumias.academy | helpdesk@forumias.academy
Page 20 of 45

PTS 2024 | B4 | L1 Test 2 - Solutions |

Statement 1 is correct: The RAMP scheme offers the scope for concerned State governments to
prepare their own plan for the development of MSME sectors in the state. Important component of
RAMP is preparation of Strategic Investment Plans (SIPs), in which all States/UTs will be invited to
prepare their development plans for MSMEs and the funding will be based on the appraisal of this plan by
the Ministry of MSME.
Statement 2 is correct: The Word Bank provides funding support for this scheme. Of the total financial
outlay of 6,062.45 crore, World bank provided 3750 crores as loans based on the fulfilment of following
conditions by the Government of India.
1) Implementing the National MSME Reform Agenda
2) Accelerating MSME Sector Centre-State collaboration
3) Enhancing effectiveness of Technology Upgradation Scheme (CLCS-TUS)
4) Strengthening Receivable Financing Market for MSMEs
5) Enhancing Effectiveness of Credit Guarantee Trust for Micro and Small Enterprises (CGTMSE) and
Greening and Gender delivery
6) Reducing the incidence of delayed payments
Statement 3 is incorrect: At national level the Ministry of Micro, Small and Medium enterprises
(MoMSME) act as a nodal agency for this scheme, not PM Economic Advisory Council. RAMP program
will help setup a high-level MSME Council, headed by the Minister for MSME to enable better
coordination between national and state-level programs and further it provides overall monitoring and
policy overview of RAMP programme.
Source: https://pib.gov.in/PressReleasePage.aspx?PRID=1811360

Q.46)
Ans) a
Exp) Option a is the correct answer.
Pair 1 is incorrect. Under Article 371-B of the constitution of India, the President is empowered to
provide for the creation of a committee of the Assam Legislative Assembly consisting of the members
elected from the Tribal Areas of the state and such other embers as he may specify.
There is no such provision in Articles 371 to 371-J regarding the special responsibility of the Governor of
Assam with respect to the law and order in the state.
Pair 2 is incorrect. The Governor of Nagaland shall have special responsibility for law and order in the
state so long as internal disturbances caused by the hostile Nagas continue. In the discharge of this
responsibility, the Governor, after consulting the Council of Ministers, exercises his individual judgement
and his decision is final. This special responsibility of the Governor shall cease when the President so
directs.
There is no such provision in Articles 371 to 371-J regarding the special responsibility of the Governor of
Nagaland with respect to the establishment of scientific institutions in the state.
Pair 3 is incorrect. Under the Article 371-F of the constitution, the Governor of Sikkim shall have special
responsibility for peace and for an equitable arrangement for ensuring the social and economic
advancement of the different sections of the Sikkim population. In the discharge of this responsibility, the
Governor shall act in his discretion, subject to the directions issued by the President.
There is no such provision in Articles 371 to 371-J regarding the special responsibility of the Governor of
Sikkim with respect to the maintenance of Political stability.
Pair 4 is correct. Under Article 371-J, the President is empowered to provide that the Governor of
Karnataka would have special responsibility for the establishment of a separate development board for
Hyderabad-Karnataka region (The Hyderabad - Karnataka region includes the six backward districts of
Northern Karnataka, viz., Gulbarga, Bidar, Raichur, Koppal, Yadgir and Bellary).
Source: laxmikant chapter 30

Forum Learning Centre: Delhi - 2nd Floor, IAPL House, 19 Pusa Road, Karol Bagh, New Delhi - 110005 | Patna - 2nd floor, AG Palace, E Boring Canal
Road, Patna, Bihar 800001 | Hyderabad - 1st & 2nd Floor, SM Plaza, RTC X Rd, Indira Park Road, Jawahar Nagar, Hyderabad, Telangana 500020
9311740400, 9311740900 | https://academy.forumias.com | admissions@forumias.academy | helpdesk@forumias.academy
Page 21 of 45

PTS 2024 | B4 | L1 Test 2 - Solutions |

Q.47)
Ans) d
Exp) Option d is the correct answer.
Statement 1 is incorrect. A person who is not a member of the state legislature can be appointed as Chief
Minister for six months, within which time, he should be elected to the state legislature, failing which he
ceases to be the Chief Minister.
Statement 2 is incorrect. The Constitution does not require that a person must prove his majority in the
legislative assembly before he is appointed as the Chief Minister. The governor may first appoint him as
the Chief Minister and then ask him to prove his majority in the legislative assembly within a reasonable
period. This has been done in a number of cases.
Statement 3 is correct. According to the Constitution of India, the Chief Minister may be a member of
any of the two houses of a state legislature. Usually, chief ministers have been selected from the lower
house (legislative assembly), but, on a number of occasions, a member of the upper house (legislative
council) has also been appointed as Chief Minister. Example: C Rajagopalachari in Madras in 1952, Morarji
Desai in Bombay in 1952 etc.
Source: laxmikant chapter 31

Q.48)
Ans) d
Exp) Option d is the correct answer.
Statement I is incorrect. The term of the Chief Minister is not fixed, and he holds office during the
pleasure of the Governor. However, this does not mean that the governor can dismiss him at any time. He
cannot be dismissed by the governor as long as he enjoys the majority support in the legislative
assembly. But, if he loses the confidence of the assembly, he must resign or the governor can dismiss him.
Statement II is correct. Article 164 of the Constitution says the Chief Minister shall be appointed by the
Governor and other Ministers shall be appointed by the Governor on the Chief Minister’s advice. It also
adds that “the Ministers shall hold office during the pleasure of the Governor”.
Source: laxmikant chapter 31
https://www.constitutionofindia.net/articles/article-164-other-provisions-as-to-ministers/

Q.49)
Ans) b
Exp) Option b is the correct answer.
The budget is based on the principle of annuality, that is, the Parliament grants money to the
government for one financial year. If the granted money is not spent by the end of the financial year,
then the balance expires and returns to the Consolidated Fund of India. This practice is known as the
‘rule of lapse’. It facilitates effective financial control by the Parliament as no reserve funds can be built
without its authorisation. However, the observance of this rule leads to heavy rush of expenditure
towards the close of the financial year. This is popularly called as ‘March Rush’
Source: Pg 22.32, Laxmikanth, chapter 22

Q.50)
Ans) b
Exp) Option b is the correct answer.
Recently, Indian Antarctic Act, 2022 has been passed. Its key objectives include having India’s own
national measures for protecting Antarctic environment, ensuring de-militarization of the region,

Forum Learning Centre: Delhi - 2nd Floor, IAPL House, 19 Pusa Road, Karol Bagh, New Delhi - 110005 | Patna - 2nd floor, AG Palace, E Boring Canal
Road, Patna, Bihar 800001 | Hyderabad - 1st & 2nd Floor, SM Plaza, RTC X Rd, Indira Park Road, Jawahar Nagar, Hyderabad, Telangana 500020
9311740400, 9311740900 | https://academy.forumias.com | admissions@forumias.academy | helpdesk@forumias.academy
Page 22 of 45

PTS 2024 | B4 | L1 Test 2 - Solutions |

getting rid of mining or illegal activities, management of growing Antarctic tourism and sustainable
development of fisheries.
Statement 1 is correct: The Indian Antarctic Act, 2022 is applicable on Indian citizens, citizens of
foreign countries and a vessel or aircraft registered in India or outside India. It is also applicable on a
company, body corporate, corporation, partnership firm, joint venture, an association of persons or any
other entity incorporated, established or registered as such under any law in force in India.
Statement 2 is incorrect: In general, drilling, dredging, excavating, collecting samples etc. of mineral
resources is not allowed. However, permission can be granted for these activities for the purposes of
scientific research or construction, repair and maintenance of an Indian station or any other
structure. Hence, permit is required.
Statement 3 is correct: Any person who intends to go to Antarctica for the purpose of commercial
fishing shall apply for a permit to the appropriate authority.
Other regulated and prohibited activities:
Regulated Activities- ‘Permit’ is required Absolute Prohibition
1) Indian expedition to Antarctica 1) Nuclear explosion or disposal of
2) Indian station in Antarctica radioactive waste material
3) Permit for vessel and aircraft entering 2) Introducing non-sterile soil
Antarctica 3) Damage, destroy or remove any historic
4) Permit for mineral resource activities site or monument or any of its part
5) Permit for introducing non-native 4) Discharge of certain specified products
animals and plants into Antarctica. or substances.
6) Permit for introducing microscopic 5) Other specified activities
organisms.
7) Permit for discharge into sea.
8) Permit for waste disposal.
9) Permit to enter protected areas.
10) Special permit for commercial fishing
in Antarctica
11) Other specified activities
Knowledge Base:
Other important provisions of the Act:
1) Background: To give effect to the provisions of Antarctic Treaty, the Protocol on Environment
Protection (Madrid Protocol) to the Antarctic Treaty and to the Convention on the Conservation of
Antarctic Marine Living Resources. India acceded to Antarctic Treaty system in 1983.
2) Central Government shall establish a Committee on Antarctic Governance and Environmental
Protection. This committee shall be chaired by the Secretary, Ministry of Earth Sciences.
3) This committee shall establish a waste classification system and waste management plans.
Important Facts about Antarctica
1) It is the southernmost continent, fifth largest in area and contains the geographic South Pole.
2) It is mostly uninhabited, except approximately 40 permanent stations established by several countries.
Maitri and Bharti are India’s research stations on the continent.
3) Native species of animals include mites, nematodes, penguins, seals and tardigrades etc. Vegetation
occurs, mostly, in the form of lichens and moss.
Source:https://prsindia.org/files/bills_acts/acts_parliament/2022/The%20Indian%20Antarctic%20Ac
t,%202022.pdf
https://pib.gov.in/PressReleasePage.aspx?PRID=1847047

Forum Learning Centre: Delhi - 2nd Floor, IAPL House, 19 Pusa Road, Karol Bagh, New Delhi - 110005 | Patna - 2nd floor, AG Palace, E Boring Canal
Road, Patna, Bihar 800001 | Hyderabad - 1st & 2nd Floor, SM Plaza, RTC X Rd, Indira Park Road, Jawahar Nagar, Hyderabad, Telangana 500020
9311740400, 9311740900 | https://academy.forumias.com | admissions@forumias.academy | helpdesk@forumias.academy
Page 23 of 45

PTS 2024 | B4 | L1 Test 2 - Solutions |

Q.51)
Ans) a
Exp) Option a is the correct answer.
The Constitution does not contain any specific procedure for the selection and appointment of the Prime
Minister. Article 75 says only that the Prime Minister shall be appointed by the President.
The Prime Minister enjoys many powers as head of the Union Council of Ministers.
Statement a is incorrect: The function of appointing any member of Parliament as a minister is not
exercised by the Prime Minister of India as the head of the Union council of ministers. The Prime
Minister recommends individuals to be appointed as ministers to the President, but the final
appointment is made by the President.
Statement b is correct: Assigning portfolios to ministers and making changes if needed is a function
exercised by the Prime Minister. The Prime Minister has the authority to assign specific portfolios to
ministers and can reshuffle them as required.
Statement c is correct: Controlling the activities of all the ministers is a function of the Prime Minister.
The Prime Minister guides, directs, controls, and coordinates the activities of all the ministers to ensure
the smooth functioning of the government.
Statement d is correct: Advising the President to dismiss any minister is a function of the Prime Minister.
The Prime Minister can ask a minister to resign or advise the President to dismiss a minister if there is a
difference of opinion or if it is deemed necessary.
Source: Laxmikanth Chapter no 19 Prime Minister

Q.52)
Ans) b
Exp) Option b is the correct answer.
Article 123 of the Constitution of India grants the President certain law-making powers to promulgate
ordinances when either of the two Houses of Parliament is not in session.
Statement 1 is incorrect: The capacity of the President to legislate through an ordinance is not a parallel
legislative power given under the Constitution. It is an extraordinary power granted to the President
under Article 123 of the Constitution. Only when either of the houses of the Parliament is not in session
then the President can promulgate an Ordinance. An ordinance passed when both Houses are in session
is null and invalid. The ordinance making power is a co-extensive power, it is not a parallel legislation
due to various safeguards in the Constitution.
Statement 2 is correct: The President ordinance-making power is co-extensive with the legislative
power of the Parliament. This means that he can issue ordinances only on those subjects on which the
Parliament can make laws.
Statement 3 is correct: President can promulgate an ordinance only when both the Houses of Parliament
are not in session or when either of the two Houses of Parliament is not in session. The second provision
implies that an ordinance can also be promulgated by the president when only one House is in session
because a law can be passed by both the Houses and not by one House alone.
Statement 4 is incorrect: If an ordinance lapses or is not re-promulgated, it does not automatically
invalidate or nullify the actions taken under that ordinance. The lapsing of an ordinance does not have a
retrospective effect on the actions or decisions that were taken during the period when the ordinance
was in force. The Supreme Court has held that actions taken while the ordinance was in effect would hold
as if the ordinance continued to exist.
Source: Laxmikanth, C-17
https://www.businesstoday.in/magazine/focus/story/budget-session-parliament-narendra-modi-
controversial-ordinances-143824-2015-01-29

Forum Learning Centre: Delhi - 2nd Floor, IAPL House, 19 Pusa Road, Karol Bagh, New Delhi - 110005 | Patna - 2nd floor, AG Palace, E Boring Canal
Road, Patna, Bihar 800001 | Hyderabad - 1st & 2nd Floor, SM Plaza, RTC X Rd, Indira Park Road, Jawahar Nagar, Hyderabad, Telangana 500020
9311740400, 9311740900 | https://academy.forumias.com | admissions@forumias.academy | helpdesk@forumias.academy
Page 24 of 45

PTS 2024 | B4 | L1 Test 2 - Solutions |

Q.53)
Ans) b
Exp) Option b is the correct answer.
The 103rd Constitutional Amendment Act provides for reservation in appointments to posts under the
state and in admissions to educational institutions to “economically weaker sections of citizens [EWS]”.
Statement 1 is correct. It applies to citizens belonging to the economically weaker sections of any sect of
society other than the Schedule Caste, Scheduled Tribes and Other Backward Classes.
Statement 2 is correct. The 103rd Constitutional amendment act have inserted article 15 (6) and 16 (6) in
the Indian constitution. These newly added articles provide for up to 10% reservation to economically
weaker sections of the society for getting admission in higher educational institutions and initial
appointment in government jobs.
Statement 3 is incorrect. The 103rd amendment act provides for reservation of jobs in central
government jobs as well as government educational institutions. It is also applicable on admissions to
private higher educational institutions (except minority educational institutions).
Source: Appendix IV Constitutional Amendments at a Glance
https://blog.forumias.com/answered-103rd-constitutional-amendment-present-a-more-difficult-
judicial-examination-than-usual-comment-discuss-103rd-amendment-act-and-its-salient-features/

Q.54)
Ans) a
Exp) Option a is the correct answer
Under Article 352, the President can declare a national emergency when the security of India or a part of
it is threatened by war or external aggression or armed rebellion.
Statement 1 is correct: It is true that the Proclamation of emergency must be approved by both houses
of the Parliament with Special majority i.e, a majority of the total membership of that house, and a
majority of not less than two-thirds of the members of that house present and voting. This special
majority provision was introduced by the 44th Amendment Act of 1978. Previously, such a resolution
could be passed by a simple majority of the Parliament.
Statement 2 is incorrect: The President can declare a National emergency even when neither of houses
of Parliament are in session, if he/she is satisfied that there is an imminent threat to the nation.
Statement 3 is incorrect: Parliament has an important role in both approval as well as extension of
National emergency. Proclamation of National emergency if approved by both the Houses of Parliament
then it continues for six months, and can be extended to an indefinite period with an approval of the
Parliament for every six months. This provision for periodical parliamentary approval was added by the
44th Amendment Act of 1978.
Source: Laxmikanth - Chapter 16 (Emergency Provisions)

Q.55)
Ans) a
Exp) Option a is the correct answer.
In the Union Budget 2023-24, millets have been referred to as ‘shree anna’. India is the leading producer
and consumer of the millets. Consumption of millets leads to food and nutritional security, in addition to
the welfare of the farmers. The Budget aims to make India a global hub of millets. On India’s initiative,
United Nations has declared the year 2023 as the ‘International Year of Millets’.
Option a is correct: All the three items- Kodo, Proso and Foxtail are the different varieties of millets.
Option b is incorrect: All the three items- lentils, chickpeas and vetches are the different varieties of
pulses.

Forum Learning Centre: Delhi - 2nd Floor, IAPL House, 19 Pusa Road, Karol Bagh, New Delhi - 110005 | Patna - 2nd floor, AG Palace, E Boring Canal
Road, Patna, Bihar 800001 | Hyderabad - 1st & 2nd Floor, SM Plaza, RTC X Rd, Indira Park Road, Jawahar Nagar, Hyderabad, Telangana 500020
9311740400, 9311740900 | https://academy.forumias.com | admissions@forumias.academy | helpdesk@forumias.academy
Page 25 of 45

PTS 2024 | B4 | L1 Test 2 - Solutions |

Option c is incorrect: Sorghum and Kodo are the millet varieties, but chickpea is a variety of pulses.
Option d is incorrect: Foxtail and Proso are the millet varieties, but vetches are a variety of pulses.
A brief account on millets:
Earliest evidence of millets is found in Indus Valley Civilization (3000 BC). Millets are a group of small
seeded grass that are grown for their grains, which are used as both for humans and animals. Millets are
good source of proteins, fiber and essential nutrients. They are also used as staple food in various parts
of the world where other grains are scarce. They are called as famine reserves as they can grow in very
little rainfall. Millets are environmentally sustainable crops as they require less water and are resistant
to pests and diseases. Different types of millets are:

(Source: Indian Express)


Source: https://krishijagran.com/agripedia/know-about-different-types-of-millet/
https://www.indiabudget.gov.in/doc/budget_speech.pdf
https://blog.forumias.com/millet-production-in-india/

Q.56)
Ans) c
Exp) Option c is the correct answer
The procedure for proclamation of National emergency and its continuation is different from that of
revocation of national emergency. Following are the procedures followed by the Government and the
Parliament to revoke National emergency.
Statement 1 is correct: The 44th Constitutional Amendment Act (CAA), 1978, made it compulsory for
the President to revoke emergency if Lok Sabha passes the resolution for the same with simple
majority. Also, a proclamation of emergency may be revoked by the President at any time and it does
not require Parliamentary approval.

Forum Learning Centre: Delhi - 2nd Floor, IAPL House, 19 Pusa Road, Karol Bagh, New Delhi - 110005 | Patna - 2nd floor, AG Palace, E Boring Canal
Road, Patna, Bihar 800001 | Hyderabad - 1st & 2nd Floor, SM Plaza, RTC X Rd, Indira Park Road, Jawahar Nagar, Hyderabad, Telangana 500020
9311740400, 9311740900 | https://academy.forumias.com | admissions@forumias.academy | helpdesk@forumias.academy
Page 26 of 45

PTS 2024 | B4 | L1 Test 2 - Solutions |

Statement 2 is correct: It is true that the State Legislatures does not have any power to revoke National
emergency. This is the exclusive power of Lok Sabha, thus State legislatures and even Rajya Sabha has no
role to play in it.
Source: Laxmikanth - Chapter 16 (Emergency Provisions)

Q.57)
Ans) c
Exp) Option c is the correct answer.
A proclamation of Emergency has drastic and wide-ranging effects on the political system. While a
proclamation of Emergency is in force, the normal fabric of the Centre-State relations undergoes some
changes.
Statement 1 is incorrect: In contrast to the President's rule, the Parliament cannot delegate the
legislative power of the State to any other authority during the operation of a National emergency.
Hence during National emergency, the Parliament can make laws on subjects enumerated in the
State List only by itself i.e., it cannot delegate the same to any other body or authority.
Statement 2 is correct: It is true that during the operation of National emergency, the Parliament can
impose duties upon the Centre and officers of Central government to carry out matters lying outside
the Union List. This provision enables the Parliament to carry out the laws made by it under its extended
jurisdiction, as a result of the proclamation of a National Emergency.
Statement 3 is correct: It is true that the President can issue ordinances on State subjects, if the
Parliament is not in session, during the operation of National emergency.
Statement 4 is correct: One of the financial implications of National emergency is that the President can
either reduce or cancel the transfer of finances from the Centre to the States.
Source: Laxmikanth - Chapter 16 (Emergency Provisions)

Q.58)
Ans) a
Exp) Option a is the correct answer
President’s Rule refers to the suspension of a state government and the imposition of direct rule of the
Centre. The central government takes direct control of the state in question and the Governor becomes
its constitutional head.
Option 1 is incorrect: The threat to financial stability of the State Government is one of the grounds for
imposition of Financial Emergency (under Article 360) in the State. Further the Supreme Court in
S.R.Bommai Case,1994 stated that stringent financial exigencies of State is not the proper ground for
imposition of President’s rule in the State.
Option 2 is correct: The Supreme Court in S.R.Bommai Case, 1994, states that President’s rule can be
imposed in States where the government wilfully refuses to discharge its constitutional obligations
endangering the security of the state.
Option 3 is correct: Article 365 says that whenever a State fails to comply with or to give effect to any
direction from the Centre, it will be lawful for the President to hold that a situation has arisen in which
the government of the state cannot be carried on in accordance with the provisions of the Constitution.
In such a scenario the President issues a proclamation under Article 356 and takes over the control of the
State government. Therefore failure to comply with the constitutional direction of the Central
government is one of the grounds for imposition of President’s rule in the State.
Option 4 and 5 are incorrect: The Supreme Court in S.R.Bommai Case, 1994 enlisted following as the
improper ground for declaration of President’s rule in the State.

Forum Learning Centre: Delhi - 2nd Floor, IAPL House, 19 Pusa Road, Karol Bagh, New Delhi - 110005 | Patna - 2nd floor, AG Palace, E Boring Canal
Road, Patna, Bihar 800001 | Hyderabad - 1st & 2nd Floor, SM Plaza, RTC X Rd, Indira Park Road, Jawahar Nagar, Hyderabad, Telangana 500020
9311740400, 9311740900 | https://academy.forumias.com | admissions@forumias.academy | helpdesk@forumias.academy
Page 27 of 45

PTS 2024 | B4 | L1 Test 2 - Solutions |

1) Maladministration in the state or allegations of corruption against the State Council of Ministers or
Stringent financial exigencies of the state.
2) If the ruling party enjoying majority support in the assembly has suffered a massive defeat in the
general elections to the Lok Sabha.
Source: Laxmikanth - Chapter 16 (Emergency Provisions)
https://www.legalserviceindia.com/legal/article-4367-article-356-president-s-rule.html

Q.59)
Ans) a
Exp) Option a is the correct answer.
The Constitution (Article 165) has provided for the office of the advocate general for the states. S/he is
the highest law officer in the state.
Statement 1 is incorrect. The term of office of the advocate general is not fixed by the Constitution.
Further, the Constitution does not contain the procedure and grounds for his removal. He holds office
during the pleasure of the governor. This means that he may be removed by the governor at any time.
He may also quit his office by submitting his resignation to the governor. Conventionally, he resigns when
the government (council of ministers) resigns or is replaced, as he is appointed on its advice.
It is the Members of the State Public Service Commission who cannot be removed by Governor, they
can be removed only by the President on the report of the Supreme Court on reference made by the
President and, in some cases, on the happening of certain disqualifications [Art. 317].
Statement 2 is correct. The Advocate general must be a person who is qualified to be appointed a judge
of a high court. In other words, he must be a citizen of India and must have held a judicial office for ten
years or been an advocate of a high court for ten years.
Statement 3 is incorrect. In the performance of his official duties, the advocate general is entitled to
appear before any court of law within the state. Further, he has the right to speak and to take part in the
proceedings of both the Houses of the state legislature or any committee of the state legislature of which
he may be named a member, but without a right to vote. He enjoys all the privileges and immunities that
are available to a member of the state legislature. [Art. 177].
Source: Indian Polity: Laxmikanth: 6th edition, Chapter 31, Page 969-70
Introduction to the Constitution of India: DD Basu, Page 133,240

Q.60)
Ans) b
Exp) Option b is the correct answer.
Mahila Samman Savings Certificate scheme is a small savings scheme backed by the government. Scheme
will give a fixed interest rate of 7.5%, much higher than most bank Fixed Deposits (FDs) and other
popular small savings schemes.
Option a is correct: The Mahila Samman Savings Certificate can be done only in the name of a girl child
or woman, unlike other small saving schemes like Public Provident Fund (PPF), National Savings
Certificate (NSC), Senior Citizens Small Savings Scheme (SCSS) and Sukanya Samriddhi Yojana (SSY)
Option b is incorrect: The maximum deposit amount under the Mahila Samman Savings Certificate is
Rs.2 lakh.
Option c is correct: This scheme is a one-time scheme with a tenure of two years. This scheme will be
available for two years, i.e., from 2023 to 2025.
Option d is correct: A partial withdrawal facility is provided under the Mahila Samman Saving
Certificate scheme. Hence, it allows for premature withdrawal of the deposits.
Source: https://cleartax.in/s/mahila-samman-saving-certificate
Union Budget Speech 2023-24

Forum Learning Centre: Delhi - 2nd Floor, IAPL House, 19 Pusa Road, Karol Bagh, New Delhi - 110005 | Patna - 2nd floor, AG Palace, E Boring Canal
Road, Patna, Bihar 800001 | Hyderabad - 1st & 2nd Floor, SM Plaza, RTC X Rd, Indira Park Road, Jawahar Nagar, Hyderabad, Telangana 500020
9311740400, 9311740900 | https://academy.forumias.com | admissions@forumias.academy | helpdesk@forumias.academy
Page 28 of 45

PTS 2024 | B4 | L1 Test 2 - Solutions |

Q.61)
Ans) c
Exp) Option c is the correct answer.
The Vice-President, like the president, is elected indirectly. He is elected by the members of an electoral
college consisting of the members of both Houses of Parliament.
Option 1 and 4 are correct: The Vice-President of India is elected by an electoral college consisting of all
the members of both houses of Parliament. This includes both Nominated members of Parliament and
elected members of Parliament. Whereas in case of President only the elected members of Parliament
participate.
Option 2 and 3 are incorrect. The elected members of State Legislative Assemblies and the nominated
members of State Legislative Council do not participate in the election of Vice President.
Knowledge Base: The electoral college of Vice President is different from the electoral college for the
election of the President in the following two respects:
1) It consists of both elected and nominated members of the Parliament
2) It does not include the members of the state legislative assemblies
Source: Laxmikanth Chapter 18 Vice President

Q.62)
Ans) a
Exp) Option a is the correct answer.
Since 1950, the President’s Rule has been imposed on more than 100 occasions, that is, on an average
twice a year. Hence, Article 356 has become one of the most controversial and most criticised provision
of the Constitution.
Manipur is the state that has witnessed President's rule the maximum number of times (ten times)
among the states of India, followed by Uttar Pradesh (Nine times). Manipur has faced political instability
and other factors leading to the imposition of President's rule on several occasions.

Source: Laxmikanth Chapter 16 Emergency


https://www.thehindu.com/news/national/how-many-times-has-presidents-rule-been-imposed-
across-states/article30017580.ece
https://factly.in/wp-content/uploads//2018/06/MHA-Presidents-Rule.pdf

Q.63)
Ans) d
Exp) Option d is the correct answer.
Article 360 empowers the president to proclaim a Financial Emergency if he is satisfied that a situation
has arisen due to which the financial stability or credit of India or any part of its territory is threatened.
Statement I is incorrect: Financial Emergency has not been declared in India since its independence in
1947. Even when there have been financial crises, such as the one in 1991, a declaration of a Financial has
not taken place.
Statement II is correct: In 1991, India encountered a significant financial crisis known as the "1991
Economic Crisis". During this time, India experienced a balance of payments crisis, characterized by a
depletion of foreign exchange reserves and a high fiscal deficit. The crisis was mainly caused by a
combination of factors such as rising oil prices, a large fiscal deficit, a balance of payments problem, and a
stagnant economy.
Source: laxmikant chapter 16 Emergency

Forum Learning Centre: Delhi - 2nd Floor, IAPL House, 19 Pusa Road, Karol Bagh, New Delhi - 110005 | Patna - 2nd floor, AG Palace, E Boring Canal
Road, Patna, Bihar 800001 | Hyderabad - 1st & 2nd Floor, SM Plaza, RTC X Rd, Indira Park Road, Jawahar Nagar, Hyderabad, Telangana 500020
9311740400, 9311740900 | https://academy.forumias.com | admissions@forumias.academy | helpdesk@forumias.academy
Page 29 of 45

PTS 2024 | B4 | L1 Test 2 - Solutions |

Q.64)
Ans) c
Exp) Option c is the correct answer.
Statement a is incorrect: Prime minister office (PMO) can be defined as the administrative agency created
under article 77 (3) in order to provide secretarial assistance. It has the status of department under
Allocation of Business rules 1961. It is accorded as extra-constitutional body and not a statutory body.
On the other hand, Cabinet Secretariat is an executive body. The Government of India (Allocation of
Business) Rules, 1961 (AoB Rules) and the Government of India (Transaction of Business) Rules, 1961 (ToB
Rules) have been framed under Article 77 (3) of the Constitution of India and Cabinet Secretariat is
responsible for administration and implementation of both these rules.
Statement b is incorrect: The Cabinet Secretariat functions directly under the Prime Minister. The
administrative head of the Secretariat is the Cabinet Secretary who is also the ex-officio Chairman of the
Civil Services Board. Whereas, PMO is a staffing agency assisting the Prime Minister of India in the
efficient discharge of his role, functions, and responsibilities. It functions directly under the Prime
Minister.
Statement c is correct: The PMO provides secretarial assistance to the Prime Minister. It is headed by
the Principal Secretary to the Prime Minister. PMO is an extra-constitutional body that has no mention in
the Indian Constitution. However, it was given the status of a department under the Government of India
Allocation of Business Rules, 1961.
On the other hand, the Cabinet Secretariat is responsible for the administration of the Government of
India (Transaction of Business) Rules, 1961 and the Government of India (Allocation of Business) Rules
1961, facilitating smooth transaction of business in Ministries/ Departments of the Government.
Statement d is incorrect: The Cabinet Secretariat keeps the President, Vice President and all the
ministries informed of the major activities of the government by circulating monthly summaries and brief
notes on important matters.
Source: Laxmikant 6th Edition.pdf
https://cabsec.gov.in/content.php?page=12
https://egyankosh.ac.in/bitstream/123456789/25784/1/Unit-8.pdf

Q.65)
Ans) c
Exp) Option c is the correct answer.
Recently, the Prime Minister of India has expressed deep grief over the demise of noted Yakshagana
playback singer, Shri Balipa Narayana Bhagawatha.
Yakshagana is a traditional folk-dance form popular in Coastal Karnataka. It is a rare combination of
dance, music, song, scholarly dialogues and colourful costumes. Traditionally, men portray all roles,
including the female ones, though women are now part of Yakshagana troupes. A typical troupe
consists of 15 to 20 actors and a Bhagawatha, who is the master of ceremonies and the main storyteller.

Forum Learning Centre: Delhi - 2nd Floor, IAPL House, 19 Pusa Road, Karol Bagh, New Delhi - 110005 | Patna - 2nd floor, AG Palace, E Boring Canal
Road, Patna, Bihar 800001 | Hyderabad - 1st & 2nd Floor, SM Plaza, RTC X Rd, Indira Park Road, Jawahar Nagar, Hyderabad, Telangana 500020
9311740400, 9311740900 | https://academy.forumias.com | admissions@forumias.academy | helpdesk@forumias.academy
Page 30 of 45

PTS 2024 | B4 | L1 Test 2 - Solutions |

Yakshagana
1) Each performance typically focuses on a small sub-story (known as ‘Prasanga’) from ancient Hindu
epics of Ramayana or Mahabharata. The show consists of both stage performances by talented artists
and commentary (performed by the lead singer or Bhagawatha) accompanied by traditional music.
2) Musical instruments used in Yakshagana include Chande (drums), Harmonium, Maddale, Taala (mini
metal clappers) and flute among others.
3) Costumes used in Yakshagana are very unique and elaborate. Large size headgear, coloured faces,
elaborate costumes all over the body and musical beads on the legs (Gejje).
Source: https://pib.gov.in/PressReleasePage.aspx?PRID=1900033

Q.66)
Ans) c
Exp) Option c is the correct answer.
The First Constitutional Amendment Act of 1951 to the Indian Constitution was brought to address certain
concerns and ensure a balance between the fundamental right to freedom of speech and expression and
the larger interests of the society and the nation.
Statement 1 is correct- The First Amendment, 1951 to the Indian Constitution introduced the Ninth
Schedule, which was created to safeguard land reform and other laws that were included in it from being
subject to judicial review. This step was taken to ensure that these laws could be implemented effectively
without the risk of being challenged in court.
Statement 2 is correct- The First Amendment to the Indian Constitution empowered the state to enact
special provisions aimed at advancing socially and economically backward classes. This amendment
granted the state the authority to adopt affirmative action measures and implement policies that address
the specific needs and challenges faced by marginalized sections of society.
Statement 3 is correct- The First Amendment to the Indian Constitution introduced additional grounds
for restricting freedom of speech and expression. These include maintaining public order, preserving
friendly relations with foreign states, and preventing incitement to an offense. Furthermore, the
amendment made these restrictions subject to the requirement of reasonableness, making them
justiciable and open to judicial scrutiny.
Statement 4 is incorrect- The Sixty-First Amendment Act, 1989 (and not the First Amendment Act, 1951)
reduced the voting age from 21 years to 18 years for the Lok Sabha and state legislative assembly
elections.
Source: Laxmikanth, 6th edition, Appendix IV Constitutional Amendments at a Glance

Q.67)
Ans) d
Exp) Option d is the correct answer.
The constitution requires the previous sanction or recommendation of president for introducing
legislation on some matters. The courts are debarred from invalidating any legislation on the ground that
previous sanction was not obtained where the president has eventually assured to the legislation (Art
255).
Following are the matters which require the previous sanction or recommendation of president:
1) A bill for the formation of new states or the alteration of boundaries, etc. of existing states (Art 3).
The exclusive power of recommending such legislation is given to the president in order to enable him
to obtain the views of the affected states before initiating such legislation. (Hence statement 1 is
correct.)
2) A bill providing for any of the matter specified in art (31A)

Forum Learning Centre: Delhi - 2nd Floor, IAPL House, 19 Pusa Road, Karol Bagh, New Delhi - 110005 | Patna - 2nd floor, AG Palace, E Boring Canal
Road, Patna, Bihar 800001 | Hyderabad - 1st & 2nd Floor, SM Plaza, RTC X Rd, Indira Park Road, Jawahar Nagar, Hyderabad, Telangana 500020
9311740400, 9311740900 | https://academy.forumias.com | admissions@forumias.academy | helpdesk@forumias.academy
Page 31 of 45

PTS 2024 | B4 | L1 Test 2 - Solutions |

3) A money bill [Art 117(1)]. (Hence statement 2 is correct.)


4) A bill which would involve the expenditure from the consolidated fund of India [Art 117(3)] (Hence
statement 3 is correct.)
5) State bill imposing the restriction upon the freedom of trade. [Art 304] (Hence statement 4 is
correct.)
6) A bill affecting taxation in which states are interested or affecting the principle laid down for
distributing moneys to the states, or the varying meaning of the expression of ‘Agricultural income’
for the purpose of taxation of income or imposing a surcharge for the purpose of the union under the
chapter 1 of part XII [Art 274(1)] (Hence statement 5 is correct.)
Source: introduction to the constitution of India by DD basu. Chapter name- The Union executive. page
no-185 and 186

Q.68)
Ans) a
Exp) Option a is the correct answer.
Option 1 is correct: Article 245 of the Constitution gives Parliament the power to make laws for the
whole or any part of India, and state legislatures the power to make laws for the state. Parliament draws
its power to repeal a law from the same provision.
Option 2 is incorrect: In the United Kingdom, all Appropriation Acts usually contain a repealing provision
which specifically repeals older Appropriation Acts. In Australia the route followed is that of automatic
repeal for Appropriation Acts.
In India, however, no such mechanism is in place and Appropriation Acts continue to sit on statute-
books. The Law Commission recommended that a practice like the one of the United Kingdom to include
a repeal clause in the Appropriation Act every year would serve a useful purpose, without necessitating
major amendments or introduction of new laws.
Option 3 is incorrect: Laws in India can be repealed in two ways — either through an ordinance, or
through legislation. In case an ordinance is used, it would need to be replaced by a law passed by
Parliament within six months. If the ordinance lapses because it is not approved by Parliament, the
repealed law can be revived.
Option 4 is incorrect: The legislation for repealing of the law have to be passed by both Houses of
Parliament. The government had brought legislation to repeal the farm laws. It will have to be passed by
both Houses of Parliament and receive the President’s assent before it comes into effect. Usually, Bills
titled Repealing and Amendment are introduced for this purpose.
Source: Explained: The process for repealing a law (forumias.com)
Explained: The Process Of Repealing A Law In The Parliament (indiatimes.com)
repeal.pdf (parliamentlibraryindia.nic.in)

Q.69)
Ans) a
Exp) Option a is the correct answer.
The Forty-Fourth Amendment Act, of 1978 aimed to restore certain fundamental rights that were
curtailed during the period of emergency in the country. This amendment strengthened the protection of
individual freedoms, including the right to life and personal liberty, by imposing restrictions on the
suspension of rights. It emphasized the importance of safeguarding democratic values and ensuring
greater accountability of the government.
Statement 1 is correct- The Forty-Fourth Amendment Act, 1978 replaced the term “internal
disturbance” with “armed rebellion” in respect of national emergency under article 352.

Forum Learning Centre: Delhi - 2nd Floor, IAPL House, 19 Pusa Road, Karol Bagh, New Delhi - 110005 | Patna - 2nd floor, AG Palace, E Boring Canal
Road, Patna, Bihar 800001 | Hyderabad - 1st & 2nd Floor, SM Plaza, RTC X Rd, Indira Park Road, Jawahar Nagar, Hyderabad, Telangana 500020
9311740400, 9311740900 | https://academy.forumias.com | admissions@forumias.academy | helpdesk@forumias.academy
Page 32 of 45

PTS 2024 | B4 | L1 Test 2 - Solutions |

Statement 2 is incorrect- The Forty-Second Amendment Act, of 1976 (and not the Forty-Fourth
Amendment Act, of 1978) provided for the creation of the all-India Judicial Service.
Statement 3 is correct- The Forty-Fourth Amendment Act, of 1978 introduced a crucial provision stating
that the fundamental rights enshrined in Articles 20 and 21 of the Constitution cannot be suspended
even during a national emergency. This amendment reinforced the protection of individual rights and
ensured that certain fundamental freedoms, such as the right against self-incrimination and the right to
life and personal liberty, remain inviolable in any emergency situation.
Statement 4 is correct- The Forty-Fourth Amendment Act, of 1978 removed the right to property from
the list of Fundamental Rights and reclassified it as a legal right. This amendment transformed the right
to property from a constitutional guarantee to a mere statutory protection, subject to the discretion of
the legislature.
Source: Laxmikanth, 6th Edition, Appendix IV Constitutional Amendments at a Glance

Q.70)
Ans) a
Exp) Option a is the correct answer
The Indian Pharmaceuticals industry plays a prominent role in the global pharmaceuticals Industry. India
is the largest provider of generic medicines globally, accounting for 20% of global supply by volume.
Statement 1 is correct: It is true that India is ranked 3rd worldwide in the production of pharma
products by volume and 14th by value. Furthermore, India is the leading vaccine manufacturer globally
with a market share of 60 per cent.
Statement 2 is incorrect: Pharmaceutical industry is not one of the eight core industries of the Indian
economy. The main or the key industries constitute the core sectors of an economy. The eight-core
industries of the Indian economy are electricity, steel, refinery products, crude oil, coal, cement,
natural gas and fertilisers.
Statement 3 is incorrect: Electricity attracted the largest share of private investment in India, followed
by Steel, Chemicals, Automobile, Mining and Pharmaceutical industry. Hence the pharmaceutical industry
ranked fifth in terms of its share of total private investments in the country.

Statement 4 is incorrect: The value of pharmaceutical exports has increased in the last five years.
Indian Pharmaceutical exports received a growth of 24 per cent in 2021 due to Covid 19 pandemic induced
demand for drugs and even after the waning of covid 19 the sector witnessed the steady increase in
exports.

Forum Learning Centre: Delhi - 2nd Floor, IAPL House, 19 Pusa Road, Karol Bagh, New Delhi - 110005 | Patna - 2nd floor, AG Palace, E Boring Canal
Road, Patna, Bihar 800001 | Hyderabad - 1st & 2nd Floor, SM Plaza, RTC X Rd, Indira Park Road, Jawahar Nagar, Hyderabad, Telangana 500020
9311740400, 9311740900 | https://academy.forumias.com | admissions@forumias.academy | helpdesk@forumias.academy
Page 33 of 45

PTS 2024 | B4 | L1 Test 2 - Solutions |

Source: Economic Survey 2022-23 : Pg no 281


https://pib.gov.in/PressReleasePage.aspx?PRID=1894918#:~:text=warned%20that%20export-,growth,-
may%20slow%20further

Q.71)
Ans) a
Exp) Option a is the correct answer.
The Constitution of India has provided (in article 102) that a member of parliament will be disqualified
from membership if:
1) He holds any office of profit under the Union or state government (except that of a minister or any
other office exempted by Parliament)
2) He is of unsound mind and stands so declared by a court.
Statement a is correct: The question regarding the disqualification of member of Parliament in all
matters except defection is decided by President. However, President should obtain the opinion of the
election commission before taking such decision.
The question of disqualification under Anti-defection is decided by the Chairman in the case of Rajya
Sabha {i.e. Vice-President} and Speaker in the case of Lok Sabha.
Statement b is incorrect: The Anti-defection law does not specify a time-period for the Presiding Officer
to decide on a disqualification plea. There have been several cases where the Courts have expressed
concern about the unnecessary delay in deciding such petitions. In some cases this delay in decision
making has resulted in members, who have defected from their parties, continuing to be members of the
House.
Statement c is incorrect: The courts can intervene only after the Presiding Officer has decided on the
matter, the petitioner seeking disqualification has no option but to wait for this decision to be made.
Statement d is incorrect: The presiding officer of a House is empowered to make rules to give effect to
the provisions of the Tenth Schedule. All such rules must be placed before the House for 30 days. The
House may approve or modify or disapprove them. Further, he may direct that any willful contravention
by any member of such rules may be dealt with in the same manner as a breach of privilege of the House.
Source: laxmikant chapter-22 pag-22.5
https://prsindia.org/theprsblog/the-anti-defection-law-explained

Q.72)
Ans) a
Exp) Option a is the correct answer.

Forum Learning Centre: Delhi - 2nd Floor, IAPL House, 19 Pusa Road, Karol Bagh, New Delhi - 110005 | Patna - 2nd floor, AG Palace, E Boring Canal
Road, Patna, Bihar 800001 | Hyderabad - 1st & 2nd Floor, SM Plaza, RTC X Rd, Indira Park Road, Jawahar Nagar, Hyderabad, Telangana 500020
9311740400, 9311740900 | https://academy.forumias.com | admissions@forumias.academy | helpdesk@forumias.academy
Page 34 of 45

PTS 2024 | B4 | L1 Test 2 - Solutions |


The recent Centre-West Bengal controversy over Political and administrative elements has brought
attention towards the federal structure in India. Department of Personnel and Training (DoPT) under
Ministry of Personnel directed that West Bengal Chief Secretary Alapan Bandyopadhyay report to its
office.
Option 1 is incorrect: The Chief Secretary is the top-most executive official and senior-most civil servant
of the state government. The appointment of the Chief Secretary of a state is the exclusive preserve of
the chief minister. The CS appointment is essentially a transfer order issued by the General
Administration Department of the state government in the name of the Governor, but it can be effectively
done only after the chief minister has made her choice.
Option 2 is incorrect: The office of Chief Secretary has been excluded from the operation of the tenure
system. There is no fixed tenure for this post. For the post of the Chief Secretary of a state, fixed tenure
of three to four years has been recommended, but neither the Union Public Service Commission nor the
DoPT or state governments has accepted this recommendation for the simple reason that a chief minister
needs to hold the administrative reins in their hands and has received a public mandate to do so.
Option 3 is correct: The Chief Secretary is the ex-officio head of the state Civil Services Board, the
State Secretariat, the state cadre Indian Administrative Service. The Chief Secretary acts as the principal
advisor to the chief minister on all matters of state administration.
Option 4 is incorrect: Chief Ministers of States used to retain trusted chief secretaries as advisers after
retirement. Recently this trend has been observed in many states including West Bengal, Andhra Pradesh,
Odisha, Maharashtra. There is no provision which states that they cannot be appointed in any office to
serve the government after retirement.
Source: Explained: Chief Secretary Appointment Controversy – TheLeaflet
Explained | Is the order attaching West Bengal Chief Secretary unprecedented? - The Hindu

Q.73)
Ans) c
Exp) option c is the correct answer.
The 104th Constitutional Amendment Act has amended Article 334 to cease the nomination of Anglo-
Indians in the Lok Sabha and the state legislative assemblies and has extended the reservation of seats for
the SC/ST community. The amendment came into effect on January 25, 2020. The Article 334 has laid
down provisions for reserving the seats for SCs and STs and nomination of Anglo-Indians to Lok Sabha
and state Assemblies, which shall cease to be in effect on the 25 January 2020, if not extended further.
The term Anglo Indian is defined as per the article 366 (2) of the Indian constitution; “a person whose
father or any of whose other male progenitors in the male line is or was of European descent but who is a
native of India.”
Source: Appendix IV Constitutional Amendments at a Glance
https://www.google.com/amp/s/www.thehindu.com/news/national/parliament-approves-10-year-
extension-to-sc-st-reservation-anglo-indian-nomination-dropped/article30289758.ece/amp/

Q.74)
Ans) d
Exp) Option d is the correct answer.
Options 1, 3 and 4 are correct: Public Account of India: All other public money (other than those which
are credited to the Consolidated Fund of India) received by or on behalf of the Government of India shall
be credited to the Public Account of India. This includes provident fund deposits, judicial deposits,
savings bank deposits, departmental deposits and remittances. This account is operated by executive

Forum Learning Centre: Delhi - 2nd Floor, IAPL House, 19 Pusa Road, Karol Bagh, New Delhi - 110005 | Patna - 2nd floor, AG Palace, E Boring Canal
Road, Patna, Bihar 800001 | Hyderabad - 1st & 2nd Floor, SM Plaza, RTC X Rd, Indira Park Road, Jawahar Nagar, Hyderabad, Telangana 500020
9311740400, 9311740900 | https://academy.forumias.com | admissions@forumias.academy | helpdesk@forumias.academy
Page 35 of 45

PTS 2024 | B4 | L1 Test 2 - Solutions |


action, that is, the payments from this account can by made without parliamentary appropriation. Such
payments are mostly in the nature of banking transactions
Option 2 is incorrect: Consolidated Fund of India is a fund to which all receipts are credited and all
payments are debited. In other words, (a) all revenues received by the Government of India; (b) all loans
raised by the Government by the issue of treasury bills, loans or ways and means of advances; and (c)
all money received by the government in repayment of loans forms the Consolidated Fund of India.
Source: Laxmikanth, chapter 22

Q.75)
Ans) b
Exp) Option b is the correct answer.
Foreign exchange reserves are required for various purposes, such as to maintain the exchange rate
within a certain range, or to maintain a certain level of exchange rate if the country follows a fixed
exchange rate system; maintain liquidity and allow time to absorb shocks.
Option a is incorrect: The optimal level of reserves empowers the government to maximise the welfare
of the representative consumer, given the constraint of sudden stops in terms of debt rollover crisis.
Therefore, adequacy of reserves comes in picture. Conventionally, there are three measures of adequacy
of reserves, viz. ratios of reserves to imports, to monetary aggregates, and measures of external debt
Option b is correct: Guidotti–Greenspan IMF rule states that a country's reserves should be equal to
short-term external debt (one-year or less maturity), suggesting a ratio of reserves-to-short term debt
of one. The traditional approach determines the adequacy of reserves by three months of imports or full
cover of short-term debt. This approach is based on Guidotti–Greenspan IMF rule.
Option c and d are incorrect: Guidotti–Greenspan IMF rule does not talk about Gross Fixed capital
formation nor it talks about the precautionary measures to be taken by the economy in economic crisis.
Source: Page 331, chapter 11:External Sector: Watchful and Hopeful of Economic survey 2022-23.

Q.76)
Ans) b
Exp) Option b is the correct answer.
Council of States i.e. Rajya Sabha is the Upper House of the Indian Parliament. Rajya Sabha consists of the
representatives of the States and the Union Territories and persons nominated by the President of India.
The Vice-President of India is the ex-officio Chairman of Rajya Sabha. The Rajya Sabha has been given
four exclusive or special powers that are not enjoyed by the Lok Sabha.
Statement 1 is incorrect: Rajya Sabha can authorise the Parliament to make a law on a subject
enumerated in the State List (Article 249). There is no such special power where Rajya Sabha has been
given power to authorise State Legislature to make a law on a subject enumerated in the union list.
Statement 2 is correct: Rajya Sabha can authorise the Parliament to create new All-India Services
common to both the Centre and states (Article 312). For that the Rajya Sabha passes a resolution
declaring that it is essential in the interest of the nation.
Statement 3 is correct: Only Rajya Sabha alone can initiate a move for the removal of the vice-
president. In other words, a resolution for the removal of the vice-president can be introduced only in
the Rajya Sabha and not in the Lok Sabha (Article 67).
Source: Laxmikant chapter 22: Parliament

Q.77)
Ans) c
Exp) Option c is the correct answer.

Forum Learning Centre: Delhi - 2nd Floor, IAPL House, 19 Pusa Road, Karol Bagh, New Delhi - 110005 | Patna - 2nd floor, AG Palace, E Boring Canal
Road, Patna, Bihar 800001 | Hyderabad - 1st & 2nd Floor, SM Plaza, RTC X Rd, Indira Park Road, Jawahar Nagar, Hyderabad, Telangana 500020
9311740400, 9311740900 | https://academy.forumias.com | admissions@forumias.academy | helpdesk@forumias.academy
Page 36 of 45

PTS 2024 | B4 | L1 Test 2 - Solutions |


The Parliament participates in the election of the President (along with the state legislative assemblies)
and elects the Vice-President. The Lok Sabha elects its Speaker and Deputy Speaker, while the Rajya
Sabha elects its Deputy Chairman.
Option 1, 2 and 3 are correct. The Parliament is also authorised to make laws to regulate the
elections to the offices of President and Vice-President, to both the Houses of Parliament and to both
the Houses of state legislature. Accordingly, Parliament enacted the Presidential and Vice-Presidential
Election Act (1952), the Representation of People Act (1950), the Representation of People Act (1951), etc.
Source: Laxmikant chapter 22: Parliament

Q.78)
Ans) c
Exp) Option c is the correct answer.
The Constitution refers to the budget as the ‘annual financial statement’. In other words, the term
‘budget’ has nowhere been used in the Constitution. It is the popular name for the ‘annual financial
statement’ that has been dealt with in Article 112 of the Constitution.
Statement 1 is correct: The President shall put before both the Houses of Parliament a statement of the
estimated receipts and expenditure of the Government of India for that year, which is referred as the
annual financial statement. As per Article 112 (2)(b), the budget shall distinguish expenditure on revenue
account from other expenditure
Statement 2 is correct: As per Article 117, Parliament can reduce or abolish a tax but cannot increase it.
However, no tax shall be levied or collected except by authority of law.
Statement 3 is correct: As per Article 114, no money shall be withdrawn from the Consolidated Fund of
India except under appropriation made by law. The appropriation bill gives power to the government to
withdraw funds from the consolidated fund of India for meeting the expenditure during the financial
year.
Source: Laxmikant chapter 22: Parliament

Q.79)
Ans) a
Exp) Option a is the correct answer.
Bills introduced in the Parliament are of two kinds: public bills and private bills (also known as
government bills and private members’ bills respectively). Though both are governed by the same general
procedure and pass through the same stages in the House, they differ in various respects.
Statement 1 is incorrect: Private bill cannot be introduced by any member of Parliament. Minister
cannot introduce private bills in the Parliament. Bills introduced by the Ministers in the Parliament are
called Public bill.
Statement 2 is correct: Rejection of Private bill by the House has no implication on the parliamentary
confidence in the government or its resignation. While rejection of Public Bill by the House reflects loss
of parliamentary confidence in the government and may lead to its resignation.
Statement 3 is incorrect: Introduction of Public Bill in the House requires only seven days’ notice.
While Introduction of Private bill in the House requires one month’s notice (not 7 days). While a Public
Bill can be introduced and discussed on any day, a private bill can only be introduced and discussed on
Fridays.
Source: Laxmikant chapter 22: Parliament
https://indianexpress.com/article/explained/private-members-bill-government-bill-parliament-
6263844/

Forum Learning Centre: Delhi - 2nd Floor, IAPL House, 19 Pusa Road, Karol Bagh, New Delhi - 110005 | Patna - 2nd floor, AG Palace, E Boring Canal
Road, Patna, Bihar 800001 | Hyderabad - 1st & 2nd Floor, SM Plaza, RTC X Rd, Indira Park Road, Jawahar Nagar, Hyderabad, Telangana 500020
9311740400, 9311740900 | https://academy.forumias.com | admissions@forumias.academy | helpdesk@forumias.academy
Page 37 of 45

PTS 2024 | B4 | L1 Test 2 - Solutions |

Q.80)
Ans) d
Exp) Option d is the correct answer.
Experts at NASA are tracking a giant asteroid named 199145 (2005 YY128) which is just days away from
entering Earth’s orbit. The asteroid is classified as a potentially hazardous asteroid (PHA) because its
orbit brings it within 0.05 astronomical units (AU) of Earth's orbit. Tracking and studying asteroids like
199145 (2005 YY128) is important for identifying potential threats to Earth.
199145 (2005 YY128): It is an asteroid whose orbit crosses the orbit of Earth. It orbits the sun every 774
days (2.12 years) in an elliptical orbit. 2005 YY128 is probably between 566 to 1.265 kilometers in diameter.
Its orbit is 0.03 astronomical unit from Earth's orbit at its closest point.
Source: https://economictimes.indiatimes.com/news/new-updates/giant-asteroid-to-hit-earths-
atmosphere-find-out-when-will-it-happen/articleshow/97797292.cms
https://www.virtualtelescope.eu/2023/02/10/potentially-hazardous-asteroid-199145-2005-yy128-
close-encounter-a-image-09-feb-2023/

Q.81)
Ans) b
Exp) Option b is the correct answer.
The first hour of every parliamentary sitting is slotted for Question Hour. During this time, the members
ask questions, and the ministers usually give answers.
Statement 1 is correct: Though during Question Hour, questions are asked mainly to the Ministers. But
there is no restrictions to ask questions to other members of Parliament than Ministers. Hence, the
questions can also be asked to any members of Parliament. A question may be addressed to a private
member if the subject matter of the question relates to some Bill, resolution or other matter connected
with the business of the House for which that member is responsible.
Statement 2 is correct: Parliamentary Rules limit the number of starred and unstarred questions that
can be asked in a day. The total number of questions asked by MPs in the starred and unstarred
categories are then put in a random ballot. From the ballot in Lok Sabha, 20 starred questions are picked
for answering during Question Hour and 230 are picked for written answers.
Statement 3 is incorrect: Question Hour was suspended during the Chinese aggression in 1962.
Parliamentary records show that during the Chinese aggression in 1962, the Winter Session was
advanced. The sitting of the House started at 12 pm and there was no Question Hour held. Before the
session, changes were made limiting the number of questions. Thereafter, following an agreement
between the ruling and the Opposition parties, it was decided to suspend Question Hour. Even during
the Monsoon session in 2020, the Lok Sabha and Rajya Sabha secretariats notified that there will be no
Question Hour during the Monsoon Session of Parliament in view of the Covid-19 pandemic.
Source: Laxmikant chapter 22: Parliament
https://indianexpress.com/article/explained/an-expert-explains-what-are-question-hour-zero-hour-
parliament-session-6580747/

Q.82)
Ans) d
Exp) Option d is the correct answer.
Option 1, 2, 3 and 4 are correct.
The ministers are collectively responsible to the Parliament in general and to the Lok Sabha in particular.
This means that they continue in office so long as they enjoy the confidence of the majority members in
the Lok Sabha. In other words, the council of ministers can be removed from office by the Lok Sabha by

Forum Learning Centre: Delhi - 2nd Floor, IAPL House, 19 Pusa Road, Karol Bagh, New Delhi - 110005 | Patna - 2nd floor, AG Palace, E Boring Canal
Road, Patna, Bihar 800001 | Hyderabad - 1st & 2nd Floor, SM Plaza, RTC X Rd, Indira Park Road, Jawahar Nagar, Hyderabad, Telangana 500020
9311740400, 9311740900 | https://academy.forumias.com | admissions@forumias.academy | helpdesk@forumias.academy
Page 38 of 45

PTS 2024 | B4 | L1 Test 2 - Solutions |

passing a no-confidence motion. The Lok Sabha can also express lack of confidence in the government
in the following ways:
1) By rejecting a money bill
2) By not passing a motion of thanks on the President’s inaugural address.
3) By defeating the government on a vital issue. (Rejection of the Constitutional Amendment Bill does
not amount to expressing of lack of confidence in the government)
4) By passing a censure motion or an adjournment motion.
5) By passing a cut motion.
Source: Pg 22.31, Laxmikanth, chapter 22

Q.83)
Ans) b
Exp) Option b is the correct answer.
Groups of Ministers(GoMs) are formed to give recommendations to the cabinet on certain emergent
issues and critical problem areas.
Option a is incorrect: Groups of Ministers(GoMs) are ad-hoc bodies. These are not permanent in
nature. Ministers heading the concerned ministries are inducted into the relevant GoMs and when the
advice is crystallised, they are disbanded.
Option b is correct: Any decision taken by a Standing or Ad hoc Committee may be reviewed by the
Cabinet’. Therefore, decisions in a matter taken by GoMs remain subject to review by the Cabinet at the
latter’s discretion.
Option c is incorrect: The 2nd ARC did not recommend that abolition of GoMs . The 2nd ARC observed that
large number of GoMs are not able to meet regularly and thus lead to delays. It also recommended that
GoMs should take decision within a prescribed time limit.
Option d is incorrect: Some of these GoMs are empowered to take decisions on behalf of the Cabinet
whereas the others make recommendations to the Cabinet.
Source: Laxmikanth 6th edition chapter-21 page- 21.2

Q.84)
Ans) b
Exp) Option b is the correct answer.
Article 360 empowers the president to proclaim a Financial Emergency if he is satisfied that a situation
has arisen due to which the financial stability or credit of India or any part of its territory is threatened.
Statement 1 is correct: A proclamation declaring financial emergency must be approved by both the
Houses of Parliament within two months from the date of its issue. However, if the proclamation of
Financial Emergency is issued at a time when the Lok Sabha has been dissolved or the dissolution of the
Lok Sabha takes place during the period of two months without approving the proclamation, then the
proclamation survives until 30 days from the first sitting of the Lok Sabha after its reconstitution,
provided the Rajya Sabha has in the meantime approved it.
Statement 2 is incorrect: Once approved by both the Houses of Parliament, the Financial Emergency
continues indefinitely till it is revoked. This implies two things:
1) there is no maximum period prescribed for its operation; and
2) repeated parliamentary approval is not required for its continuation.
Statement 3 is correct: A proclamation of Financial Emergency may be revoked by the president at any
time by a subsequent proclamation. Such a proclamation does not require the parliamentary approval.
Source: Laxmikanth chapter 16 Emergency

Forum Learning Centre: Delhi - 2nd Floor, IAPL House, 19 Pusa Road, Karol Bagh, New Delhi - 110005 | Patna - 2nd floor, AG Palace, E Boring Canal
Road, Patna, Bihar 800001 | Hyderabad - 1st & 2nd Floor, SM Plaza, RTC X Rd, Indira Park Road, Jawahar Nagar, Hyderabad, Telangana 500020
9311740400, 9311740900 | https://academy.forumias.com | admissions@forumias.academy | helpdesk@forumias.academy
Page 39 of 45

PTS 2024 | B4 | L1 Test 2 - Solutions |

Q.85)
Ans) b
Exp) Option b is the correct answer.
Union Finance Minister recently announced the Central Government’s plan to launch a mission to
eliminate sickle cell anemia by 2047.
Statement 1 is correct: Sickle cell anemia is an inherited blood disorder that affects the production of
hemoglobin, a protein in red blood cells that carries oxygen throughout the body. In individuals with
sickle cell anemia, the hemoglobin protein is abnormal, causing red blood cells to become rigid, sticky,
and crescent-shaped (resembling a sickle) instead of the normal round shape. These abnormal red blood
cells can get stuck in small blood vessels, reducing blood flow and causing various complications.
Statement 2 is correct: Sickle cell disease is an autosomal recessive genetic disorder, which means that a
person needs to inherit two copies of the abnormal hemoglobin gene (one from each parent) in order
to develop the disease.
If both parents have the sickle cell trait (one copy of the abnormal gene and one normal gene), there is a
25% chance that each of their children will inherit two copies of the abnormal gene and develop sickle
cell disease, a 50% chance that each child will inherit one copy of the abnormal gene and be a carrier of
the trait, and a 25% chance that each child will inherit two normal genes and neither have the disease nor
be carriers of the trait.
Statement 3 is incorrect: Sickle cell anemia cannot be prevented through vaccination as it is a genetic
disorder. The only cure for this disease is bone marrow or stem cell transplantation.
However, there are treatments that can help relieve symptoms, lessen complications, and prolong life.
Source: https://economictimes.indiatimes.com/industry/healthcare/biotech/healthcare/health-
ministrys-fight-against-sickle-cell-anaemia-to-follow-gujarat-playbook/articleshow/97531998.cms
https://pib.gov.in/PressReleasePage.aspx?PRID=1896039#:~:text=Sickle%20cell%20disease%20(SCD)%2
0is,significant%20reduction%20in%20life%20expectancy.
https://www.pib.gov.in/PressReleseDetailm.aspx?PRID=1902841

Q.86)
Ans) a
Exp) Option a is the correct answer.
Article 88 of the Indian Constitution provides for the Rights of Ministers as Respects the Houses.
Every minister shall have the right to speak in and to take part in the proceedings of either House, any
joint sitting of the Houses.
Statement 1 is correct: Every minister shall have the right to take part in the proceedings of either
House. Every minister shall have the right to take part in the proceedings of any joint sitting of the
Houses.
Statement 2 is incorrect: Every minister shall have the right to take part in the proceedings of any
Committee of Parliament of which he may be named a member. If he is not a member of the
Parliamentary Committee, he cannot take part in its proceedings.
Statement 3 is incorrect: A minister who is a member of one House of Parliament has the right to speak
and to take part in the proceedings of the other House also, but he can vote only in the House of which
he is a member.
Source: Indian Polity by Laxmikant 6th Edition – Chapter 20 Central Council of Ministers.

Q.87)
Ans) d
Exp) Option d is the correct answer.

Forum Learning Centre: Delhi - 2nd Floor, IAPL House, 19 Pusa Road, Karol Bagh, New Delhi - 110005 | Patna - 2nd floor, AG Palace, E Boring Canal
Road, Patna, Bihar 800001 | Hyderabad - 1st & 2nd Floor, SM Plaza, RTC X Rd, Indira Park Road, Jawahar Nagar, Hyderabad, Telangana 500020
9311740400, 9311740900 | https://academy.forumias.com | admissions@forumias.academy | helpdesk@forumias.academy
Page 40 of 45

PTS 2024 | B4 | L1 Test 2 - Solutions |

Statement 1 is incorrect. When the office of the President falls vacant due to unforeseen circumstances
like death, etc, the Vice President only acts as the President for the intermittent period, until someone
else is duly elected as the President. The Vice President does not succeed to the office for the remaining
term when it falls vacant.
Statement 2 is incorrect. All doubts and disputes in connection with election of the Vice- President are
inquired into and decided by the Supreme Court whose decision is final. The election of a person as Vice-
President cannot be challenged on the ground that the electoral college was incomplete (i.e., existence of
any vacancy among the members of electoral college).
Statement 3 is incorrect. If the election of a person as Vice President is declared void by the Supreme
Court, acts done by him before the date of such declaration of the Supreme Court are not invalidated
(i.e., they continue to remain in force).
Source: http://164.100.47.5/Chairman-Rajyasabha/VPElection.htm
Indian Polity by Laxmikant – Vice President.

Q.88)
Ans) c
Exp) Option c is the correct answer.
Statement a is correct. The Constitution under article 267(1) authorised the Parliament to establish a
‘Contingency Fund of India’ into which amounts determined by law are paid from time to time.
Accordingly, the Parliament enacted the contingency fund of India Act in 1950.
Statement b is correct. Contingency fund of India is operated by executive action, that is, the
payments from this account can be made without parliamentary appropriation.
Money from consolidated fund of India cannot be appropriated (issued or drawn) except in accordance
with a parliamentary law.
Statement c is incorrect A Contingency Fund of India which is an account placed at the disposal of the
President of India to facilitate meeting of urgent unforeseen expenditure by the Government. The corpus
of the Contingency Fund as authorized by Parliament was 500 crore. In the Union Budget of India 2021-
22, a proposal to enhance the corpus of the Contingency Fund of India from ₹500 crore to ₹30,000 crores
was introduced.
Statement d is correct: The fund is held by the finance secretary on behalf of the president.
Source: https://www.indiabudget.gov.in/budget2019-20(I)/ub2019-20/keybud/keybud2019.pdf
Indian polity, M Laxmikanth. Page no.22.30

Q.89)
Ans) c
Exp) Option c is the correct answer
Each House of Parliament has its own presiding officer. There is a Speaker and a Deputy Speaker for the
Lok Sabha and a Chairman and a Deputy Chairman for the Rajya Sabha.
Statement 1 is correct: While the Speaker of Lok Sabha is elected by the Members of Lok Sabha from
amongst them, the Vice President of India is the ex-officio Chairperson of the Rajya Sabha. Therefore, it
is true that the Chairperson of Rajya Sabha is not elected by the members of Rajya Sabha.
Statement 2 is correct: When the resolution for removal of Speaker/Chairperson is under consideration,
the Chairperson cannot vote at first instance, but he/she can be present and speak in the House and
can take part in its proceedings. The Speaker can vote in the first instance when a resolution for his/her
removal is under consideration.

Forum Learning Centre: Delhi - 2nd Floor, IAPL House, 19 Pusa Road, Karol Bagh, New Delhi - 110005 | Patna - 2nd floor, AG Palace, E Boring Canal
Road, Patna, Bihar 800001 | Hyderabad - 1st & 2nd Floor, SM Plaza, RTC X Rd, Indira Park Road, Jawahar Nagar, Hyderabad, Telangana 500020
9311740400, 9311740900 | https://academy.forumias.com | admissions@forumias.academy | helpdesk@forumias.academy
Page 41 of 45

PTS 2024 | B4 | L1 Test 2 - Solutions |

Statement 3 is correct: Speaker of the Lok Sabha can be removed from his/her office, if the Lok Sabha
passes a resolution supported by a majority of all then members of the Lok Sabha, thus Rajya Sabha has
no role in removing the Speaker of Lok Sabha.
Chairperson of the Rajya Sabha is removed from his/her office, if the Rajya Sabha passes a resolution
supported by a majority of all the then members of the Rajya Sabha. However, such a resolution has to be
supported by the Lok Sabha (with simple majority), thus Lok Sabha plays a role in removing the
Chairperson.
Source: Laxmikanth - Chapter 22 (Parliament)

Q.90)
Ans) b
Exp) Option b is the correct answer.
Government officials informed that 20 Russian banks have opened Special Rupee Vostro Accounts (SRVA)
with partner banks in India.
A vostro account is an account that domestic banks hold for foreign banks in the former’s domestic
currency, in this case, the rupee. Domestic banks use it to provide international banking services to their
clients who have global banking needs. It is an integral offshoot of correspondent banking that entails a
bank to facilitate wire transfers, conduct business transactions, accept deposits and gather documents on
behalf of the other bank.

Source: https://blog.forumias.com/explained-vostro-accounts-and-how-they-facilitate-trade/
https://www.thehindu.com/business/Economy/explained-vostro-accounts-and-how-they-facilitate-
trade/article66537848.ece#:~:text=A%20vostro%20account%20is%20an,who%20have%20global%20ban
king%20needs.

Q.91)
Ans) c
Exp) Option c is the correct answer.
The Governor has to exercise his powers and functions with the aid and advise of the council of ministers
headed by the chief minister, except in matters in which he is required to act in his discretion (i.e.,
without the advice of ministers).
Statement 1 is correct. While recommending the imposition of the President’s rule in a state, the
Governor can exercise his discretion.
Statement 2 is correct. The governor of a state can exercises his discretion in the case of seeking
information from the Chief Minister with regard to the administrative and legislative matters of the state.

Forum Learning Centre: Delhi - 2nd Floor, IAPL House, 19 Pusa Road, Karol Bagh, New Delhi - 110005 | Patna - 2nd floor, AG Palace, E Boring Canal
Road, Patna, Bihar 800001 | Hyderabad - 1st & 2nd Floor, SM Plaza, RTC X Rd, Indira Park Road, Jawahar Nagar, Hyderabad, Telangana 500020
9311740400, 9311740900 | https://academy.forumias.com | admissions@forumias.academy | helpdesk@forumias.academy
Page 42 of 45

PTS 2024 | B4 | L1 Test 2 - Solutions |

Statement 3 is correct. The Governor of a state can exercises discretionary powers in the case of
appointment of chief minister when no party has a clear-cut majority in the state legislative assembly or
when the chief minister in office dies suddenly and there is no obvious successor.
Statement 4 is incorrect. The Governor's ordinance making power is not a discretionary power. This
means that he can promulgate or withdraw an ordinance only on the advice of the council of ministers
headed by the Chief Minister.
Source: laxmikant chapter 30

Q.92)
Ans) b
Exp) Option b is the correct answer
The Central Government takes over the control of the State Government under Article 356 in case of
failure of constitutional machinery in the State. This is popularly known as President’s Rule and also
known as State Emergency or Constitutional Emergency.
Option 1 is incorrect: Unlike the Proclamation of National Emergency, the President’s rule does not have
any impact on the Fundamental rights mentioned in the Constitution. Thus, suspension of fundamental
rights is not a consequence of the President's rule in the State.
Option 2 is correct: The imposition of the President’s rule in the State results in either the suspension or
dissolution of the State Legislative Assembly.
Option 3 is correct: On the Proclamation of President’s rule, the State Council of Ministers is dismissed.
The President administers the state through the Governor thus the executive power of the State
government is assumed by the Central government. Dismissal of the State Council of Ministers is a
necessary consequence of the President’s rule.
Option 4 is incorrect: Dissolution of the Local bodies is not the necessary consequence of imposition of
President’s rule in the State.
Source: Laxmikanth - Chapter 16 (Emergency Provisions)

Q.93)
Ans) d
Exp) Option d is the correct answer.
A vote of credit is a financial grant provided to the executive to satisfy financial requirements, the
contents of which cannot be disclosed in the budget.
Statement 1 is incorrect: A vote of credit is a grant which is allowed for meeting an unexpected
demand upon the resources of India. It is granted when there is indefinite character of the service or the
demand cannot be stated with the details ordinarily given in a budget. While Vote on Account is a grant
given in advance to enable the government to carry on its business ceaselessly.
Statement 2 is incorrect: Vote-of-credit can be used by both regular and caretaker government. As
vote of credit is granted for meeting an unexpected demand upon the resources of India. Hence, it is like
a blank cheque given to the Executive by the Lok Sabha.
Source: Laxmikant, chapter 22: Parliament

Q.94)
Ans) c
Exp) Option c is the correct answer.
In the scheme of parliamentary system of government provided by the Constitution, Chief Minister is the
real executive authority (de facto executive). The Constitution does not contain any specific procedure

Forum Learning Centre: Delhi - 2nd Floor, IAPL House, 19 Pusa Road, Karol Bagh, New Delhi - 110005 | Patna - 2nd floor, AG Palace, E Boring Canal
Road, Patna, Bihar 800001 | Hyderabad - 1st & 2nd Floor, SM Plaza, RTC X Rd, Indira Park Road, Jawahar Nagar, Hyderabad, Telangana 500020
9311740400, 9311740900 | https://academy.forumias.com | admissions@forumias.academy | helpdesk@forumias.academy
Page 43 of 45

PTS 2024 | B4 | L1 Test 2 - Solutions |


for the selection and appointment of the Chief Minister. Article 164 only says that the Chief Minister shall
be appointed by the governor. The powers and functions of Chief Minister includes:
Statement 1 is correct: Chief Minister of a State acts as a vice-chairman of the concerned zonal council
by rotation, holding office for a period of one year at a time, while Union home minister is the chairman
of all the zonal councils.
Statement 2 is incorrect: Civil Services Board (CSB) is responsible for transfers and postings of the
Indian Administrative Service (IAS) officers in the state. The CSB is headed by the Chief Secretary of a
state and has senior most additional chief secretary or chairman, Board of Revenue, Financial
Commissioner or an officer of equivalent rank and status as member. The Chief Minister is not a
member of the Civil Services Board. This function is usually carried out by the administrative machinery.
Chief Minister is the chairman of the State Planning Board.
Statement 3 is correct: The Chief Minister of a state is a member of the Inter-State Council, which is
headed by the Prime Minister of India. The Inter-State Council facilitates coordination between the
Union and the states on matters of common interest.
Statement 4 is correct: Chief Minister plays an important role in managing crises and emergencies within
the state and act as a crisis manager-in-chief at the political level during emergencies.As a leader of the
state, he meets various sections of the people and receives memoranda from them regarding their
problems, and so on.
Source: Laxmikanth

Q.95)
Ans) b
Exp) Option b is the correct answer.
Statement 1 is incorrect and Statement 2 is correct: River Cities Alliance (RCA) is a joint initiative of
the Ministry of Jal Shakti and the Ministry of Housing and Urban Affairs. The RCA has been launched
with the objective to provide the member cities with a platform to discuss and exchange information on
aspects that are vital for sustainable management of urban rivers, sharing best practices and
supporting innovation
Statement 3 is correct: The Alliance is open to all river cities of India. Any river city can join the
Alliance at any time. State Governments are implementing Action Plans for restoration of water quality
of the identified polluted river stretches. The implementation is being monitored regularly at State level
by Chief Secretary of the respective State/UT and at Central level by the Central Monitoring Committee
under the Chairmanship of Secretary, Ministry of Jal Shakti.
Knowledge Base: The River Cities Alliance has been launched initially with 30 cities namely Dehradun,
Rishikesh, Haridwar, Srinagar, Varanasi, Kanpur, Prayagraj, Farrukhabad, Mirzapur, Mathura, Bijnor,
Ayodhya, Patna, Bhagalpur, Begusarai, Munger, Sahibganj, Rajmahal, Howrah, Jangipur, Hugli-Chinsurah,
Behrampore, Maheshtala, Aurangabad, Chennai, Bhubaneshwar, Hyderabad, Pune, Udaipur and
Vijaywada.
Source: https://pib.gov.in/PressReleasePage.aspx?PRID=1795103

Q.96)
Ans) c
Exp) option c is the correct answer.
The 24th amendment to the Constitution of India made it compulsory for the president of India to give
his assent to a Constitutional Amendment Bill which is passed by both the houses of Parliament.

Forum Learning Centre: Delhi - 2nd Floor, IAPL House, 19 Pusa Road, Karol Bagh, New Delhi - 110005 | Patna - 2nd floor, AG Palace, E Boring Canal
Road, Patna, Bihar 800001 | Hyderabad - 1st & 2nd Floor, SM Plaza, RTC X Rd, Indira Park Road, Jawahar Nagar, Hyderabad, Telangana 500020
9311740400, 9311740900 | https://academy.forumias.com | admissions@forumias.academy | helpdesk@forumias.academy
Page 44 of 45

PTS 2024 | B4 | L1 Test 2 - Solutions |


The 24th Amendment of the Indian Constitution amended Article 368 to expressly state that Parliament
has the power to amend any provision of the Constitution. The amendment further made it obligatory for
the President to give his assent when a Constitution Amendment Bill was presented to him.
Source: Appendix IV Constitutional Amendments at a Glance
https://primelegal.in/2022/11/06/a-critical-analysis-of-the-indian-constitution-twenty-fourth-
amendment-act-
1971/#:~:text=Twenty%20Fourth%20Amendment%20Act%2C%201971%3A&text=The%20Constitution%2
0(24th%20Amendment)%20Act,fundamental%20rights%2C%20would%20be%20overturned.

Q.97)
Ans) b
Exp) Option b is the correct answer.
A structured committee system was introduced in 1993 to provide for greater scrutiny of government
functioning by Parliament. Most committees of Parliament include MPs from both the Lok Sabha and
Rajya Sabha. One such example is Joint Parliamentary Committee (JPC).
Statement 1 is correct. A Joint Parliamentary Committee (JPC) is an ad-hoc body. It is set up for a specific
object and duration.
Statement 2 is correct. Joint committees are set up by a motion passed in one house of Parliament and
agreed to by the other.
Statement 3 is incorrect. The JPC recommendations are not binding upon the government. The
government may accept it or launch a fresh investigation based on the report. The government must
report on the follow-up action based on which the committee submits the action taken report in the
Parliament.
Source: https://prsindia.org/theprsblog/jpc-vs-pac

Q.98)
Ans) b
Exp) Option b is the correct answer.
A vacancy in the office of the President of India can occur in any of the following cases:
1) Expiry of tenure of five years
2) Resignation
3) Removal by Impeachment
4) Death
5) Otherwise, for example, when he is disqualified, or his election is declared void
Statement 1 is incorrect. If the office of the President falls vacant by resignation, removal, death or
otherwise, then the election to fill the vacancy shall be held within six months from the date of the
occurrence of such a vacancy.
Statement 2 is correct. From the time the newly elected President assumes charge of his office, he serves
a full term of five years.
Source: M. Laxmikanth Indian Polity Chapter-17 (Page 407)

Q.99)
Ans) b
Exp) Option b is the correct answer.
The President, however, can proclaim a national emergency only after receiving a written
recommendation from the cabinet. Article 352 defines the term ‘Cabinet’ as the council consisting of the
Prime Minister and other ministers of the Cabinet rank.

Forum Learning Centre: Delhi - 2nd Floor, IAPL House, 19 Pusa Road, Karol Bagh, New Delhi - 110005 | Patna - 2nd floor, AG Palace, E Boring Canal
Road, Patna, Bihar 800001 | Hyderabad - 1st & 2nd Floor, SM Plaza, RTC X Rd, Indira Park Road, Jawahar Nagar, Hyderabad, Telangana 500020
9311740400, 9311740900 | https://academy.forumias.com | admissions@forumias.academy | helpdesk@forumias.academy
Page 45 of 45

PTS 2024 | B4 | L1 Test 2 - Solutions |

This means that the emergency can be declared only on the concurrence of the cabinet and not merely
on the advice of the Prime Minister. In 1975, the then Prime Minister, Indira Gandhi advised the
President to proclaim emergency without consulting her cabinet. The cabinet was informed of the
proclamation after it was made, as a fait accompli. The 44th Constitutional Amendment Act of 1978
introduced this safeguard to eliminate any possibility of the prime minister alone taking a decision in this
regard.
Source: Laxmikanth

Q.100)
Ans) d
Exp) Option d is the correct answer.
Statement 1 is correct: Volatile organic compounds are compounds that have a high vapor pressure
and low water solubility. Many VOCs are human-made chemicals that are used and produced in the
manufacture of paints, pharmaceuticals, and refrigerants.
Statement 2 is correct: VOCs are carbon-containing chemicals released by petrol and diesel vehicles.
They impact air quality and human health.
Statement 3 is correct: VOCs can have a natural origin as well. Plants emit these chemicals to attract
pollinators, defend themselves from pests and predators and adapt to environmental stress.
Statement 4 is correct: Benzene belongs to a class of chemicals called volatile organic compounds, so
called because they evaporate in the air. Benzene is made from coal and petroleum sources and is present
in gasoline.
Source: https://www.downtoearth.org.in/news/pollution/electric-vehicles-can-lower-emissions-of-
volatile-organic-compounds-in-india-by-2030-study-81488
https://www.epa.gov/indoor-air-quality-iaq/what-are-volatile-organic-compounds-
vocs#:~:text=Air%20Quality%20(IAQ)-
,What%20are%20volatile%20organic%20compounds%20(VOCs)%3F,paints%2C%20pharmaceuticals%2C
%20and%20refrigerants.

Forum Learning Centre: Delhi - 2nd Floor, IAPL House, 19 Pusa Road, Karol Bagh, New Delhi - 110005 | Patna - 2nd floor, AG Palace, E Boring Canal
Road, Patna, Bihar 800001 | Hyderabad - 1st & 2nd Floor, SM Plaza, RTC X Rd, Indira Park Road, Jawahar Nagar, Hyderabad, Telangana 500020
9311740400, 9311740900 | https://academy.forumias.com | admissions@forumias.academy | helpdesk@forumias.academy

You might also like